Главная Настройка Mobile Контакты NSFW Каталог Пожертвования Купить пасскод Pics Adult Pics API Архив Реквест доски Каталог стикеров Реклама
Доски


[Ответить в тред] Ответить в тред



<<
[Назад][Обновить тред][Вниз][Каталог] [ Автообновление ] 558 | 67 | 153

Тред тупых вопросов №80 Аноним 19/11/17 Вск 04:48:06  361165  
image.png (1635Кб, 1920x1080)
image.png (697Кб, 768x576)
image.png (701Кб, 841x601)
image.png (917Кб, 900x599)
Тред вопросов о жизни, Вселенной и всем таком.

Спрашиваем то, за что в других местах выдают путёвку в биореактор. Здесь анонимные ученые мирового уровня критически рассмотрят любые гениальные идеи и нарисованные в Paint схемы.

Q: Можно быстрее?
A: Можно упасть в пузырь альбукерке, наса уже почти надула его.

Q: Я начитался охуительных историй про уфологию, че делать, нам жопа?
A: Да, тебе жопа, можешь сгонять в зогач или куда оттуда пошлют.

Q: Что будет с человеком в вакууме без скафандра / если он упадет на черную дыру / попробует ступить на поверхность газового гиганта/солнца?
A: Он умрёт.

Предыдущий тут: >>359099 (OP)
https://2ch.hk/spc/res/359099.html
Аноним 19/11/17 Вск 06:30:25  361178
>>361165 (OP)
>упасть в пузырь альбукерке
Ебанный стыд...
Во-первых, Алькубьерре.
Во-вторых, не упасть, а создавать вокруг корабля изнутри (иначе кина не будет).
В-третьих, НАСА искривляет пространство на десятимиллионную часть, контролируя это сверхточными интерферометрами, до самого варп-привода здесь - как до Антарктиды раком.
Аноним 19/11/17 Вск 07:47:03  361182
Hungry2CHungryH[...].png (346Кб, 629x473)
>>361178
> Во-первых, Алькубьерре.
> Не знать про Альбукерке
> Не понимать шуток
Аноним 19/11/17 Вск 08:00:58  361183
Есть космонавт в открытом космосе, и у него в обеих руках по АК-47, и за спиной помимо скафандорового ранца, ещё и мешок с гильзами, подключёнными к АК-47 как к пулемёту.
Итак, наш космонавт спокойно себе вращается на НОО. Вопрос в том, сколько нужно произвести выстрелов вперёд из двух рук так:
1) чтобы погасить энергию вращения частично и начать падать, ныряя в атмосферу и замедляться аэроторможением
2) чтобы погасить энергию вращения полностью и начать падать строго вниз, на поверхность
Аноним 19/11/17 Вск 08:15:07  361184
image.png (392Кб, 600x467)
>>361183
>мешок с гильзами, подключёнными к АК-47 как к пулемёту.
>picrelated
Проиграл.
Отвечая на твой вопрос - во-первых, его закрутит, если будет вперед стрелять. Надо под себя (либо где там у него центр масс).
Во-вторых, он и так упадёт в атмосферу с орбиты МКС (400км): http://www.lizard-tail.com/isana/lab/orbital_decay/
Упадет и сгорит через 650 дней.
В-третьих, не хватит характеристической скорости, чтобы погасить орбитальную скорость, если только мешок патронов.
Орбиту примем за МКСовскую 7.66 км/с
>Дульная энергия пули — 2030 Дж (207 кгс·м)
Энергию пули за 2кДж
Массу космонавта за 100кг
А эту задачку оставлю на читателя.
Аноним 19/11/17 Вск 08:30:42  361185
>>361184
Да, не пули, а патроны.
> Отвечая на твой вопрос - во-первых, его закрутит, если будет вперед стрелять. Надо под себя (либо где там у него центр масс)
Я учёл вращательный момент, но не знал, как написать, что, мол, стрельба проходила через плоскость "АК-центр_тяжести-АК". Ну вот, написал, перечитал и убедился, что прописать толком не могу.
> Упадет и сгорит через 650 дней.
Наверное, стоило сказать, максимально быстро.
> если только мешок патронов
Ну это гипотетически легко.
Аноним 19/11/17 Вск 09:02:10  361186
>>361185
>Наверное, стоило сказать, максимально быстро.
Это если ты погасишь орбитальную скорость. А еще при этом будешь ускоряться по направлению к поверхности.

Я, кстати, давно не огурцировал, забыл, что быстрее - остановиться на орбите или опустить перигей в поверхность и где - на другой стороне или на четверти орбиты? Вроде все же свободное падение быстрее или сопоставимо если пренебречь атмосферой, да?
Аноним 19/11/17 Вск 10:30:38  361187
Если будет конец света - отправят ли нас сычевать на Марс?
Аноним 19/11/17 Вск 12:01:03  361189
Анон, привет. Я не силен в космической физике-механике, но у меня возник вопрос тут.
Возьмем гипотетически бесконечный, неограниченный ничем источник энергии и поставим его на космический корабль. Корабль находится на Луне и стартует в сторону Земли (это чтобы были понятны начальные условия). Я так понимаю,что скорость удаления от Луны будет равна скорости приближения к Земле, если изначально отбросить силу гравитации? Так вот. Пусть он летит со скоростью 60км в час прямо к Земле.
Может ли он с постоянной неизменной скоростью подлететь на поверхность Земли? Даже если вьебет ахулиард своих двигателей на бесконечном топливе. Или есть какое-то физическое ограничение в космосе, что он не сможет лететь меньше, чем с какой-то скоростью ?
По простому - можно ли подлететь к Земле и приземлиться на нее, не превышая скорость 60км/ч относительно Земли на всём промежутке полета, даже при входе в атмосферу?
Аноним 19/11/17 Вск 12:13:16  361191
>>361187
Зависит от вида конца света, но в 99.99% ответ:
Нет, мы все умрем.
Аноним 19/11/17 Вск 12:15:36  361193
>>361189
Да, можно, если у тебя с импульсом нет проблем.
Ты можешь это просимулировать запустив огурцач и включив там бесконечное топливо.
В начале пути тебе придется газовать, чтобы поддерживать скорость удаляясь от луны, с уменьшением ее влияния и увеличением влияния земли газовать придется слабее. В определенный равновесный момент, когда силы притяжения равны корабль развернется и начнет попукивать в сторону земли и чем ближе, тем сильнее.
Но да, физика такие маневры не запрещает.
Аноним 19/11/17 Вск 12:43:07  361196
le.png (11Кб, 1225x491)
>>361189
Скорость относительно Земной системы отсчёта у тебя (дохрена) ещё когда ты сидишь на Луне. Хотя скорость именно приближения к ней нулевая, да. Вот ты (красный крестик) сидишь на Луне, тонкие чёрные круги это орбиты Луны и тебя. Газуешь по стрелочке ровно в сторону Земли. И у тебя получается такая орбита, как на (2). Скорость приближения к Земле не очень большая, но есть ещё скорость "вбок", которая передалась тебе непосредственно от Луны (ведь ты был на орбите Луны когда сидел на Луне, сечешь?). То есть скорость газа к Земле + скорость от Луны равно (2). Чтобы упасть на Землю тебе надо будет газануть влево (против часовой стрелки) и погасить скорость, равную скорости орбитального движения Луны. Потом на тебя уже действует практически только притяжение Земли и ты падаешь на Землю (3) — чтобы не приближаться быстрее 60кмч придётся иногда оттормаживаться двигателями, чем ближе к Земле, тем чаще. Так постепенно и упадёшь. Алсо сама Луна за это время окажется уже хрен знает где. Улетит, короче, далеко. А если ты собрался весь путь лететь с такой скоростью, то лететь будешь охренеть как долго. Пнятненько?
Аноним 19/11/17 Вск 12:46:35  361197
>>361196
> чтобы не приближаться быстрее 60кмч придётся иногда оттормаживаться двигателями, чем ближе к Земле, тем чаще.
Ага, мне собственно это и нужно было.
> А если ты собрался весь путь лететь с такой скоростью, то лететь будешь охренеть как долго. Пнятненько?
Это не так важно. Просто тут в бэ пытались мне сказать, что ебать я лох, и так не получится принципиально из-за космической механики.
>>361193
>>361196
Спасибо за ответ, спейсаны!
Аноним 19/11/17 Вск 12:52:48  361198
>>361197
>и так не получится принципиально из-за космической механики
Получиться-то получится, просто, как ты и сам понимаешь, никто и никогда не будет так делать. В общем, ответ на основной вопрос: да, а вот ответ на самый первый вопрос — фиг знает. Ты вряд ли сможешь поддерживать скорость приближения к Земле 60кмч И скорость удаления от Луны 60кмч одновременно. Хотя если у тебя прямо охренеть какой умный компьютер и действительно бесконечное топливо, то он сможешь непрерывно газовать так, чтобы держать тебя ровно на линии между Луной и Землей и сделать так, чтобы ты и удалялся, и приближался с одной и той же скоростью. А если этим не заморачиваться и всего лишь поддерживать скорость приближения 60кмч — так получится, но сама Луна улетит хер знает куда и будет удаляться от тебя быстрее 60кмч. Вот.
Аноним 19/11/17 Вск 12:57:53  361199
>>361182
>Не знать классический ответ на классическую пасту
Аноним 19/11/17 Вск 15:18:56  361225
Как не думать о потенциальной сверхновой, в которую превратится hr 4210 и о великом аттракторе?
Аноним 19/11/17 Вск 15:19:49  361226
>>361225
hr 8210*, конечно же
Аноним 19/11/17 Вск 15:21:21  361227
>>361225
Берёшь и не думаешь, хуле. И вообще с этим в /psy/.
Аноним 19/11/17 Вск 15:21:55  361228
>>361225
Там еще полно водорода. Лучше подумай о том, как дожить до бабаха.
А аттрактор вообще фигня. Ну летят галактики куда-то, ну пусть и летят, миллиарды лет на это надо
Аноним 19/11/17 Вск 16:35:27  361230
>>361225
>великом аттракторе
А чё в нём такого штрашного та? Ну упадем на кучу галактик, ну будет одна мега галактика массой и размерами в стиле IC1101 Ну может на порядок, джва больше по массе даже если зародится пара квазаров по пути, поджарится некой космической цивилке будет проблематично.
Аноним 19/11/17 Вск 18:00:39  361238
>>361230
Мы вообще до аттрактора не долетим. Вселенная быстрее расширяется, чем Млечный путь летит к нему
Аноним 19/11/17 Вск 18:03:15  361241
МКС корректирует положение так, чтобы всегда быть повёрнутой одной стороной к поверхности Земли, или нет?
Аноним 19/11/17 Вск 18:04:35  361243
>>361241
Да. У нее есть надир и зенит.
Аноним 19/11/17 Вск 18:32:53  361248
>>361243
То есть там постоянно работают маневровые двигатели? На это разве не нужно дохуя топлива?
Аноним 19/11/17 Вск 18:34:13  361249
>>361248
А как тебе такая идея, что станцию разок закрутили и она продолжила вращаться вокруг своей оси с периодом в 90 минут?
Аноним 19/11/17 Вск 18:35:54  361250
>>361249
Действительно, затупил. Спасибо
Аноним 19/11/17 Вск 19:39:04  361262
>>361249
Стыковаться же неудобно
Аноним 19/11/17 Вск 21:24:17  361283
>>361262
@
ЗАКРУЧИВАЕШЬ СОЮЗ
@
ПИСОЕШЬ КАКОЕШЬ
Аноним 19/11/17 Вск 23:09:56  361287
Что-то не могу найти тему о Луне. Почему такой темы нет?
Аноним 19/11/17 Вск 23:11:51  361288
cf006194bd.png (29Кб, 720x517)
А как в 19веке узнали, что Нептун синий и буйный, чтобы дать ему такое название?
Аноним 19/11/17 Вск 23:30:24  361289
>>361288
Почитай историю названия, прежде, чем нести околесицу.
Аноним 19/11/17 Вск 23:33:09  361290
>>361289
Я почитал, там не написано. Вообще удивительна интуиция древних на этот счёт. Про габариты и влияние Юпитера они ж тоже ничего не знали.
Аноним 19/11/17 Вск 23:34:15  361291
>>361287
Она относительно неплохо изучена и там сейчас только два с половиной аппарата, а вопросы можно задать и здесь. Слишком уныло для отдельного треда, поэтому таковой никто не запилил.
>>361288
Ты натянул сову на глобус в стиле "как тыщу лет назад узнали что Венера такая горяченькая, чтобы дать ей такое название".
Аноним 19/11/17 Вск 23:35:13  361292
>>361288
Цвет бы прекрасно виден. Не зря кто-то там предложил название Океан.
Аноним 19/11/17 Вск 23:43:02  361293
А если б вас заставили удалить из сс уран или нептун, что бы вы выбрали? Какая планета лучше?
Петросян Аноним 19/11/17 Вск 23:48:38  361294
-67185996377950[...].jpg (88Кб, 1022x1024)
>>361293
>А если б вас заставили удалить из сс уран или нептун, что бы вы выбрали?
Лично я бы выбрал выезд бригады психиатров.
>Какая планета лучше?
Очевидный Нептун >>150798. Хотя подожди, я ещё не всё:
>А если б вас заставили удалить из сс уран или нептун, что бы вы выбрали?
Я бы удалил из сспц Ur Anus.
Аноним 20/11/17 Пнд 00:58:50  361299
14354846277701.jpeg (139Кб, 1280x632)
>>361294
Аноним 20/11/17 Пнд 03:14:10  361304
Tz3-XnmGFqw.jpg (44Кб, 480x675)
Анон, смотри. Есть два условия в дано: планета обитаемая и безатмосферная.
Интересно, как ты считаешь при каких условиях такое может быть.
Аноним 20/11/17 Пнд 03:16:13  361305
>>361304
конолизировали, пидоры
Аноним 20/11/17 Пнд 03:20:59  361306
>>361304
Экстремальные микробы, вероятно где то под Землёй, льдом, ну канализация. Остальные варианты в сей реальности уже не научны.
Аноним 20/11/17 Пнд 03:23:09  361307
>>361305
>>361306
А какие-нибудь кристаллы смогут выкристаллизоваться на поверхности?
Аноним 20/11/17 Пнд 03:23:38  361308
>>361306
>канализация
ПАДЛИАИМ НА ВСЕЛЕННУЮ ВСЕЙ БИОСФЕРОЙ
Аноним 20/11/17 Пнд 03:28:01  361310
>>361307
нет, без атмосферы на поверхности ничего не выкристаллизуется
Аноним 20/11/17 Пнд 03:28:37  361311
>>361307
Ну какие нибудь может и выростят, хотя для роста кристаллов нужны химические реакции, которые не очень то хорошо в вакууме на поверхности такой планеты будут происходить.

На счёт могут ли они быть живыми то тут хз, скорее всего эта вселённая не поддерживает такой контент, впрочем детально ещё не изучили...
Аноним 20/11/17 Пнд 03:29:59  361312
>>361311
> очень то хорошо в вакууме
совсем никак
Аноним 20/11/17 Пнд 03:30:16  361313
>>361310
А под поверхностью?
Аноним 20/11/17 Пнд 03:32:13  361314
>>361312
А что нибудь совсем простое? Например кристаллизация в "снежинки" продуктов извержения вулкана (Ио) или Криовулкана (Тритон)?
Аноним 20/11/17 Пнд 03:34:17  361315
>>361314
Энцелад вон водой плюётся, которая вроде как остывать в конечном итоге до состояния льда должна, хоть и постоянно испаряясь, после чего выпадать в осадок.
Аноним 20/11/17 Пнд 03:37:06  361318
>>361313
может
Аноним 20/11/17 Пнд 06:58:01  361325
2DCA51F70000057[...].jpg (39Кб, 634x357)
>>361304
Аноним 20/11/17 Пнд 07:08:50  361326
>>361325
Разве Энцедал безатмосферный?
Аноним 20/11/17 Пнд 08:03:29  361327
>>361326
Ну, смотря что считать атмосферой. По Энцеладу данных о давлении не нашел, но с Европой (у которой тоже океан) давление составляет ~750 пикометров ртутного столба. Радиус атома ртути, кстати, ~150 пикометров. Это можно считать атмосферой?
Аноним 20/11/17 Пнд 08:03:42  361328
>>361290
Юпитер был самой яркой планетой после Венеры, даже древним было понятно что эта хуйня довольно важная
Аноним 20/11/17 Пнд 08:16:06  361329
>>361327
У него атмосфера, тащемта, больше, чем на Европе
> Атмосфера Энцелада очень разреженная, но по сравнению с атмосферами других небольших спутников Сатурна — довольно плотная. В ней 91 % составляет водяной пар, 4 % — азот, 3,2 % — углекислый газ, 1,7 % — метан. Гравитации этого маленького спутника не хватает для удержания атмосферы, следовательно, есть постоянный источник её пополнения. Таким источником могут быть мощные гейзеры или криовулканы.
Аноним 20/11/17 Пнд 08:21:07  361330
>>361329
Без данных по давлению все равно неясно, что значит "плотная". Тем более, сравнение идет не с Европой а с мелкими спутниками Сатурна. По сравнению с некоторыми объектами и у Луны есть "довольно плотная" атмосфера.
Аноним 20/11/17 Пнд 08:23:22  361331
>>361294
Проиграл.

>>361304
Без проблем. Есть тут у нас одна планета, атмосферы почти нет. Полезных ископаемых найденных нет. Воды жидкой нет. Растительности нет. Населена роботами.
Аноним 20/11/17 Пнд 08:30:29  361332
15104769543971.png (428Кб, 453x604)
А может ли быть общая атмосфера для двух тел?
Например, какая-нибудь система двух тел, аналогичная Плутону с Хароном, находящаяся не там, на задворках СС, а где-нибудь в район Марса или ближе к Земле (но без учёта влияния других тел СС), от того, что солнышко-то припекает на этой орбите сильнее, отчего тела разогреваются куда больше, но, при этом, так как Плутон и Харон очень маленькие, выходящие газы - в сравнении с объёмами прочих тел и планет СС - отлетают куда дальше от поверхности. В итоге, получается, что атмосфера концентрируется не вокруг Плутона или Харона, а вокруг общего центра масс и выглядит ос тороны так, что есть некой ультраразряженное газовое тело, в центре которого вокруг общего центра вращается два не равных ядра. Возможно ли такое?
Аноним 20/11/17 Пнд 08:34:59  361333
>>361332
Что за анус на пике?
Аноним 20/11/17 Пнд 08:37:03  361334
>>361333
Сколопедра со сколопендрышами в стакане
Аноним 20/11/17 Пнд 09:03:46  361335
>>361332
они быстро упадут друг на друга
Аноним 20/11/17 Пнд 09:14:13  361336
>>361328
>Юпитер был самой яркой планетой после Венеры
Спейсач. Конец 2017 года. Мдауж.
Аноним 20/11/17 Пнд 09:27:45  361337
https://nplus1.ru/news/2017/11/15/false-vacuum
Как же всё сложно то... И вот это всё реально должно происходить. Ну почему не на что нельзя опереться, даже пустое пространство ни черта не стабильно и может кардинально поменяться, кажется что даже на стабильность законов физики нельзя рассчитывать ибо то всякие Большие взрывы происходят, то должны всякие Большие разрывы случится.. Вот как мне вилкой адекватно размышлять существуя в этом мире, если тут нет ничего неизменного и статичного? Нельзя даже ничего оставить спокойно на несколько миллиардов лет ибо ВСЁ ИЗМЕНИТСЯ. Где состояния покоя блдб?

https://nplus1.ru/news/2017/04/07/eht--sagittarius-a
Анализ этих данных по прежнему ведётся надеюсь?
Аноним 20/11/17 Пнд 10:24:38  361339
>>361337
Является вакуум ложным, или нет - вопрос открытый. Более того, если даже какая-то частица, нагло и вероломно перескочит барьер, упадёт в более низкое энергетическое состояние, воспользовавшись ничтожной вероятностью этого события, то ждать прихода можно долго. Сомневаюсь, что это случится за сараем села Пиздюкино Вологодской области. Или в нашей галактике. Мгновенно Вселенная наизнанку не вывернется. А значит будет время ещё пожить и посрать на дваче.
По второму вопросу-да, куда им деваться. Грант-же пропить освоить надо, а то следующий не дадут. Сохраняйте терпение.
Аноним 20/11/17 Пнд 10:39:00  361340
>>361336
А что не так?
Аноним 20/11/17 Пнд 10:47:12  361341
image.png (53Кб, 1462x287)
>>361340
Да тупит он. Юпитер вторая по яркости планета.
Аноним 20/11/17 Пнд 11:13:56  361342
>>361291
>Она относительно неплохо изучена и там сейчас только два с половиной аппарата

Что за аппараты? Действующие?
Аноним 20/11/17 Пнд 11:21:00  361344
image.png (7Кб, 1224x64)
>>361342
Никто не знает, информации об этом в интернете нет.
Аноним 20/11/17 Пнд 11:25:33  361345
>>361344
> Никто не знает

Так не бывает.

> Orbiter

Это не совсем то, что хотелось. Точнее - совсем не то. На поверхности действующие аппараты есть? Насколько я знаю, последний был Китай и миссия проработала гораздо меньше, чем ожидалось.
Аноним 20/11/17 Пнд 11:26:32  361346
>>361332
Ебать ужас
Аноним 20/11/17 Пнд 11:28:10  361347
>>361345
>Так не бывает.
Я думал, что не бывает того, чтобы в такой явный сарказм не могли, а однако ж!
>Это не совсем то, что хотелось. Точнее - совсем не то.
Видишь ли, в чём дело. Мало ли, что тебе хочется. На луне 2 действующих аппарата, и оба орбитеры. Всё.
>Насколько я знаю, последний был Китай и миссия проработала гораздо меньше, чем ожидалось.
Чейндж вырубился с ровером года два назад.
Аноним 20/11/17 Пнд 11:36:57  361348
>>361347
> На луне 2 действующих аппарата, и оба орбитеры. Всё.

Я категорически против употребления "на Луне" к орбитерам. Это вводит в заблуждение. Для орбитетеров следует употреблять "на лунной орбите" или "на орбите Луны".

Кстати, что за порода под слоем реголита?
Если порода скальная, то какая её прочность?
Состав?
Насколько однороден химический состав реголита в разных местах? А породы?

Если Луна неплохо изучена, то ответы на эти вопросы известны.

Аноним 20/11/17 Пнд 11:40:17  361349
>>361348
Ну, там, короче, есть серебро, литий и гелий-3.
Аноним 20/11/17 Пнд 11:41:39  361350
image.png (62Кб, 447x702)
>>361348
>Я категорически против употребления "на Луне" к орбитерам. Это вводит в заблуждение. Для орбитетеров следует употреблять "на лунной орбите" или "на орбите Луны".
Ладно, на селеноцентрической орбите, сойдёт?

>Кстати, что за порода под слоем реголита?
Зависит от местности. Там где собирали, собирали в основном базальты.


>Если порода скальная, то какая её прочность?
>Состав?
пик
>Насколько однороден химический состав реголита в разных местах? А породы?
Разнообразен, это ж не цельный шар.
Аноним 20/11/17 Пнд 12:08:45  361353
Спейсаны, обьясните для тупого, на пальцах, только без ссылок на заумь, что такое гравистар и с чем его едят? Это аналог ЧД или как?
Аноним 20/11/17 Пнд 12:23:59  361354
1024px-RobertZu[...].jpg (124Кб, 1024x1027)
>>361350

Вот это лицо на фото - враг человечества, американский инженер и публицист, основатель Марсианского общества. Он такой же враг как и все те, которые считают что начинать надо с Марса, а не с Луны.

А дело в том, что Марс далеко. Очень далеко по земным меркам. И пока ни у кого на Земле нет технологий, чтобы отправить людей на Марс. Любая миссия с экипажем обречена на провал и надолго отобьёт желание к дальним космическим полётам. Это как на гребной лодке пытаться переплыть океан. А Луна - вот она, рядом, до неё почти можно дотянуться. Люди на ней бывали.


Аноним 20/11/17 Пнд 12:47:45  361356
>>361354
в марсотред, шизик
Аноним 20/11/17 Пнд 12:53:06  361358
1[1].jpg (166Кб, 869x598)
>>361356
>в марсотред,

Куда, к фанатикам? Рассказывать что рано думать о Марсе и получить в ответ тонны фекалий? Вот -----> >>361287

Я ничего не имею против изучения Марса автоматическими станциями. Но говорить всерьёз об экспедиции на Марс в ближайшее время может только ребёнок.

И что с Луна-25? Больная тема?
Аноним 20/11/17 Пнд 12:59:54  361361
>>361358
> Куда, к фанатикам?
да, к твоим друзяшкам
это ттв, а не твой личный тред охуительных историй про врагов человечествах и прочих голосов в твоей голове
Аноним 20/11/17 Пнд 13:05:39  361363
>>361361
>да, к твоим друзяшкам

Что с Луна-25? (Только не Википедии) Что на самом деле с этим проектом, в какой он стадии и будет ли завершён?

На тебе глупые вопросы, как ты хотел.
Аноним 20/11/17 Пнд 13:11:26  361364
>>361363
на платиновые вопросы про луну-гроб-глоб с радостью ответит наш филиал >>360594 (OP)
Аноним 20/11/17 Пнд 13:20:52  361365
>>361332
Lexx 3.01 Fire and Water
Аноним 20/11/17 Пнд 13:21:13  361366
>>361364
>>361358
https://3dnews.ru/957157
http://kosmolenta.com/index.php/project-lunar/project-lunar-25
http://www.zakupki.gov.ru/epz/order/notice/ep44/view/common-info.html?regNumber=0995000000216000212
Аноним 20/11/17 Пнд 13:22:40  361367
image.png (308Кб, 356x1103)
>>361353
>Это аналог ЧД или как?
Это либо аналог (то есть, если гипотетически гравастары существуют, это не отменяет того, что могут так же параллельно существовать черные дыры)
Но изначально модель гравастара создавалась, как замена концепции черной дыры, с ее сингулярностями и прочими охуительными историями несостыковками с квантовой механикой.
Гравастар учитывает квантомеханические эффекты и являет собой сверхплотный объект физической материи, а если точнее - конденсат Бозе-Эйнштейна, тогда как черная дыра с квантммехом не дружит и является областью пространства-времени. Кроме того, если обмазаться гравистаром с ног до головы и поплыть дальше, то гравистар может объяснить темную энергию.
Аноним 20/11/17 Пнд 13:29:50  361369
>>361366

> http://www.zakupki.gov.ru/epz/order/notice/ep44/view/common-info.html?regNumber=0995000000216000212

Спасибо!
Я аналогичный вопрос задал в теме Роскосмоса, может быть инсайд подкинут.
Аноним 20/11/17 Пнд 13:31:41  361370
>>361367
> то гравистар может объяснить темную энергию.
звёзды тёмной энергии же
хотя шо то хуета шо это хуета
Аноним 20/11/17 Пнд 15:00:41  361378
>>361367
а цвет у нее черный или ИНДИГО?
Аноним 20/11/17 Пнд 18:18:42  361390
>>361378
У черной дыры какой цвет? Вот и у этой поебени такой же.
Аноним 20/11/17 Пнд 18:29:58  361392
>>361390
Зависит от температуры. Если нагреть до 4000 то черная дыра будет белого цвета, очевидно же.
Аноним 20/11/17 Пнд 18:55:51  361399
>>361392
Ты ведь троллируешь сейчас?
Аноним 20/11/17 Пнд 20:43:46  361411
Господа, как относиться вот к этой статейке?
http://dicelords.narod.ru/rockets/rocket3w.html#nostealth
Шарящие могут подтвердить или опровергнуть сей опус?
Аноним 20/11/17 Пнд 21:33:47  361418
Берем 2 объекта, которые летят на столкновение со скоростью 99% сокрости света, в точке соприкосновения создаём искривление пространства, что их скорости превысили бы скорость света - что будет?
Или я совсем поехал?
Аноним 20/11/17 Пнд 21:37:21  361419
>>361418
>что их скорости превысили бы скорость света
Не превысят. Ты ведь если просто перед объектом на 99%с искривишь пространство он же не начнёт двигаться со скоростью 198%с. Будет лететь с такой же скоростью, только пространства перед ним будет меньше и со стороны покажется, что летит быстрее света.
Аноним 20/11/17 Пнд 21:44:12  361421
>>361419
Хм, а если их разогнать до 99% в сильно замедленном пространстве, а потом хлобысь?
Аноним 20/11/17 Пнд 21:49:29  361423
>>361421
Когда ты разгоняешься до 99%с время уже начинает по-другому течь. Если дальше разгонятся, быстро долетишь до конца вселенной, превысив скорость света хоть миллион раз. Но снаружи будет казаться, что ты летишь медленнее света, потому что время по-разному течёт. Поэтому на любые твои манипуляции с хлобысями дядюшка Эйнштейн покажет фигу. Там всё само себя уравновешивает и скорость света не перехитрить.
Аноним 20/11/17 Пнд 21:51:28  361424
>>361423
Е=МЦ^2, значит что частица света не имеет массы, так? Но я слышал что имеет.
Аноним 20/11/17 Пнд 22:17:21  361428
>>361424
Не имеет. Какой-то там импульс у неё есть, но не масса.
Аноним 20/11/17 Пнд 22:19:45  361429
>>361428
Ладно, спасибо. Пойду в биореактор.
Аноним 20/11/17 Пнд 23:38:10  361467
>>361411
Там много всего, что конкретно? Да, космический корабль хуй спрячешь. Он излучает, сука.
Аноним 21/11/17 Втр 00:00:57  361484
>>361424
Это уравнение для энергии покоя. У света ее действительно нет, как и массы, но он никогда и не покоится. Зато у него всегда есть импульс, который и дает фактическую энергию. У тел с массой есть энергия покоя и, опционально, импульс, который зависит от системы отсчета.
Аноним 21/11/17 Втр 04:41:09  361572
image.png (3Кб, 196x39)
>>361424
Вот полная формула.
Аноним 21/11/17 Втр 05:37:37  361574
Что за хуй этот Карл Саган? Че он сделал такого что все на него дрочат? Это как у нас на Пушного фапать из-за того, что он Галилео вел? Или он физик дохуя? Но никаких формул или явлений его именем не названо же.
Аноним 21/11/17 Втр 06:05:21  361575
>>361467
Меня смущает достоверность формул, и теоретизирование о лёгком поиске в течении часа, ну и вытекающие отсюда дистанции. Статью почитать так астероиды искать вообще делов то, фигли они ж докуя греются, однако же мы нашли много, но далеко не все за долгие годы наблюдений
Аноним 21/11/17 Втр 06:20:33  361576
tumblrov6rdc6fb[...].gif (3060Кб, 400x300)
Короче, сколько времени будет падать камень Земли?
Берём астероид миниатюрных размеров - от 3 до 10 метров. Помещаем его на НОО и он там крутится - ладно, не прям уж НОО, об атмосферу не тормозится, но очень близко к Земле с точки зрения орбиты.
Берём второй такой же камушек, или МКС или любую другую помеху, из-за которой каменюга сходит с орбиты и начинает падать. Вопрос такой - как долго она будет падать?
Я понимаю, что в зависимости от приложения вектора и силы удара, камень может отклониться от траектории и с, для примера, строго экваториальной нормали перейти на другой плоскость, или вообще в частном случае начать вращаться строго в обратном направлении орбиты. Но меня интересует именно вариант сонаправленного движения. Как бы указать границы интересующих меня траекторий падения? Короче, мне в голову лезут синусы и косинусы, поэтому как-то так: если 1 - стабильная (вообще или на протяжении пары десятков/сотен витков) орбита, а 0 - траектория падения по прямой, перпендикулярной поверхности, то интересует интервал падения тела для [0; 0,9(9)). Так как химсостав влияет на массу, а, как я научен, в вакууме перо, камушек и подшипник падают с одинаковой скоростью, то любыми физическими параметрами, кроме формы, видимо, можно пренебречь. А форма пускай будет стандартная астеоридная - что-то типа картошки.
Аноним 21/11/17 Втр 07:10:30  361578
Сколько примерно людей в мире, с которыми хоть раз прореагировало солнечное или другое нейтрино? Несколько сотен? А два нейтрино это уже совсем перебор?
Аноним 21/11/17 Втр 10:05:07  361585
>>361578
абсолютно все
к концу жизни количество "реагирований" идёт на десятки-сотни тысяч
Аноним 21/11/17 Втр 10:23:58  361586
>>361578
Всего несколько сот кубометров льда регулярно ловят нейтрино. Значит, и человек будет реагировать с какой-то периодичностью. Их там триллионы в секунду через твоё тело проходят, алло.
Аноним 21/11/17 Втр 10:27:19  361588
>>361586
> Значит, и человек будет реагировать с какой-то периодичностью.
примерно 4 раза в сутки
Аноним 21/11/17 Втр 10:35:17  361589
xw1007555.jpg (14Кб, 720x409)
>>361575
>Статью почитать так астероиды искать вообще делов то,
Астероидов открыто около 750 000 штук. В последние годы их открывают в год несколько десятков тысяч.

>фигли они ж докуя греются,
Да не то что бы. По сравнению с тем количеством энергии, что должен излучать корабль, так вообще ни о чем.

Знакомься с пикрелейтед:
Вояджер. 40 лет. 23 ватта.
Аноним 21/11/17 Втр 10:41:21  361590
>>361589
Благодарю, яркий зараза)
Аноним 21/11/17 Втр 10:48:33  361592
>>361588
Если бы четыре раза в сутки, то не парились бы с огромными детекторами из килотонн льда а просто налили бы центнер воды и поставили датчик.
Аноним 21/11/17 Втр 10:57:40  361593
>>361574

>>361574
>Что за хуй этот Карл Саган?
Астроном, астрофизик, популяризатор науки, общественный деятель.
>Че он сделал такого что все на него дрочат?
Изучал тела солнечной системы, климат на них, возможность возникновения жизни. Популяризировал науку.

>Это как у нас на Пушного фапать из-за того, что он Галилео вел?
Видишь ли, Галилео и все прочие научно-популярные передачи растут ногами из его Космоса. Он принципиально перевернул представление о том, как должна выглядеть научно-популярная педерача, или фильм.
>Или он физик дохуя? Но никаких формул или явлений его именем не названо же.
Во-первых ещё не вечер, во-вторых область, в которой он работал не подразумевает открытия принципиально новых явлений.
Аноним 21/11/17 Втр 12:23:06  361595
>>361589
А как это он там светится ? Че происходит вобще? Он же за дохулиард км. от нас? Там лампочки или че? Поясняйте быстра! И да я тупой даун из /b, но разобраться все же надо.
Аноним 21/11/17 Втр 12:40:50  361596
>>361592
пространственное разтешение, дебич
Аноним 21/11/17 Втр 12:47:11  361597
>>361595
Там ритег стоит, тепловой генератор
Аноним 21/11/17 Втр 12:56:16  361600
>>361595
Это радио передатчик. Там джве антенны. Направленная и не направленная. Мощность излучения всего 28 или 23 ватта, в зависимости от диапазона. Вот на пике как раз это излучение и зафиксировано, как если бы ты мог видеть источники радиоволн. Естественно, для регистрации такой небольшой мощности с такого расстояния нужна большая принимающая антенна, но так-то DSN со своими тарелками уже 60 лет существует.
Аноним 21/11/17 Втр 14:51:44  361615
>>361165 (OP)
Если Юпитер на изичах притягивает в себя кучу небесных тел, то чего он не искажает свет далёких звёзд?
Аноним 21/11/17 Втр 15:00:47  361618
>>361615
Искажает, просто эффект слабый.
Вот не так давно Юпитером радиоволны от квазара гнули.
https://arxiv.org/abs/astro-ph/0302294
Аноним 21/11/17 Втр 15:05:19  361619
>>361615
чтобы оптическое что-то увидеть, нужно далече от Юпитера быть в фокальной точке.
https://www.physicsforums.com/threads/jupiter-as-a-gravitational-lens.101614/
тут какой-то ноунейм посчитал, что нужно быть на расстоянии 5500 астрономических единиц от Юпитера.
Аноним 21/11/17 Втр 16:11:15  361622
>>361618
> and tangential (retarded) deflection
Всегда подозревал, что ученые просто дебичи.
Аноним 21/11/17 Втр 17:04:03  361625
>>361597
>>361600
Двощ образовательный, пасиб. А то я думал там лампочки горят. Теперь понятней. Добра.
Аноним 21/11/17 Втр 20:05:43  361636
Когда хуббл сойдет с орбиты? Его ж не заправляют, он тормозиться должен.
Аноним 21/11/17 Втр 20:11:58  361637
>>361636
> он тормозиться должен.
не особо
планируют в 2030 свести, но это не точно
Аноним 21/11/17 Втр 20:16:49  361638
бамп
Аноним 21/11/17 Втр 21:01:22  361643
>>361622
Содомит!
Аноним 21/11/17 Втр 21:07:45  361644
>>361619
>нужно быть на расстоянии 5500 астрономических единиц от Юпитера.
Это в 1000 раз дальше, чем Юп от Солнца. Тогда его и не увидишь.
Аноним 21/11/17 Втр 22:12:26  361653
>>361644
Вот тогда-то он как раз и не будет мешаться.
бадум-тсс
Аноним 21/11/17 Втр 22:43:37  361657
>>361653
Аноним 21/11/17 Втр 22:45:13  361658
>>361657
недостаточно мыльна!
Аноним 21/11/17 Втр 22:52:49  361659
15112934177881.jpg (90Кб, 811x1040)
>>361658
Аноним 21/11/17 Втр 22:54:44  361660
>>361659
вот теперь будет чем бампать жуня-тред
Аноним 21/11/17 Втр 22:59:28  361661
Посоны, поясните. Вот есть у нас дохуя быстрый космический корабль который летит со скоростью 0,9 c. Ну, если летит мимо Земли, то и хуй с ним. Но предположим, что летит он где-нибудь в охуенно далекой галактике в направлении от нас. А галактика такая охуенно далекая, что за счет расширения пространства удаляется от нас со скоростью 0,5 c. Какая тогда будет скорость корабля относительно нас?
Аноним 21/11/17 Втр 23:50:40  361666
>>361661
там уже будет работать обычное векторное сложение скоростей и если скорость будет больше c - кораблек будет ненаблюдаем
Аноним 21/11/17 Втр 23:59:05  361667
>>361666
С хренов ненаблюдаем если свет будет лететь с такой же скоростью? По-твоему, если он удаляется со скоростью 0.9с, то свет до нас будет лететь на скорости 0.1с и минус 0.5с будет -0.4с? В галактике со скоростью удаления 0.5 свет будет долетать на скорости 0.5с. Точнее, на скорости с, но пространство расширяется и ну ты понял. Ненаблюдаем он будет только когда вылетит за границы сферы наблюдения.
Аноним 22/11/17 Срд 00:13:09  361669
>>361666
Сейчас бы складывать релятивисткие скорости, как в школе на матеше в задаче про пункт А и Б.
Аноним 22/11/17 Срд 00:20:57  361670
>>361669
Ну так вопрос в том, как же их складывать в данном случае, когда одна скорость это расширение пространства. Ну или так, пусть галактика вообще от нас удаляется со скоростью 2c. Ну вот такая вот ненаблюдаемая галактика выдалась. Как тогда складывать?
Аноним 22/11/17 Срд 00:30:06  361672
>>361670
Откуда ты знаешь, что она существует?
Аноним 22/11/17 Срд 00:36:22  361673
>>361670
Нахрен складывать если она и так ненаблюдаема?
Вообще всё просто. Объект удаляется от нас на скорости 0.9с. Пространство расширяется таким образом, что расстояние до объекта растёт дополнительно со скоростью 2с. Результаты: свет от объекта летит к нам со скоростью с, но никогда не долетит потому что пространство расширяется процентарно быстрее, чем свет успевает покрыть; объект удаляется от нас на скорости 0.9с, но расстояние до него растёт со скоростью 2.9с потому что пространство расширяется. Что сложного-то блжад?
Аноним 22/11/17 Срд 00:36:25  361674
>>361672
А какая разница? Наука не отрицает даже возможной бесконечности Вселенной. В таком случае (если считать пространство равномерно расширяющимся, на что все и указывает), должны быть галактики на любом удалении от нас, и удаляющиеся с любой, сколь угодно большой скоростью.
Аноним 22/11/17 Срд 00:40:22  361675
>>361669
> Сейчас бы не знать область применения релятивистского сложения скоростей
Аноним 22/11/17 Срд 00:44:48  361676
>>361673
Ну тут интересует именно пограничное состояние. То есть галактику 0,5 c мы видим, хуйню в ней, удаляющуюся еще на 0,49 c мы видим, а на 0,51 c уже нет? При "нормальном релятивистском" сложении получается что-то типа 0,9 + 0,9 = 0,98 (лень точные формулы искать). За счет нелокальности действительно такая разница, когда 0,9 + 0,9 = 1,8, получается?
Аноним 22/11/17 Срд 00:54:06  361678
>>361676
да, хуйня которя убегает по хабблу складывается ровно
Аноним 22/11/17 Срд 02:37:49  361688
сап,ваще нихуя почти не шарю,но кое-что интересует,а именно:
как из НИХУЯ получилась вселенная?
как из НИХУЯ произошел этот "большой взрыв" ??
да я тупой
Аноним 22/11/17 Срд 02:45:52  361689
>>361688
Хуй знает.
Аноним 22/11/17 Срд 02:58:42  361691
>>361688
Никто не знает, вообще это самое нихуя перед большим взрывом поставлено по дефолту тупо лишь потому что нихуя это самая маленькая доп сущность из всех остальных, Оккам ещё её кое как через жопу переваривает в отличие от "этих ваших" богов и прочего. В качестве альтернативы предлагают ещё циклическую генерацию вселенной по "теория нейм" (их очень не много то этих теорий с "вечной" цикличностью) но как не странно в любой из них уже больше сущностей чем у "нихуя перед большим взрывом"
Аноним 22/11/17 Срд 03:02:29  361692
>>361691
Но зато у нас достаточно данных что бы заявлять о том, что сам большой взрыв (во всяком случае все его стадии кроме самых самых начальных) все таки был.
Аноним 22/11/17 Срд 12:19:52  361708
>>361399
Я чёт не задумывался, а какую температуру имеют чёрные дыры? Различные их элементы так сказать.
Аноним 22/11/17 Срд 12:32:49  361710
>>361708
неизмеримую(кроме диска)
Аноним 22/11/17 Срд 12:51:26  361713
Подвезли очередную симуляцию формирования планетной системы, аж с 2 мя звездами (по началу)
https://youtu.be/nCfOvqeyBas
Аноним 22/11/17 Срд 14:06:26  361726
>>361710
Не пизди. Не сложно вычисляется через массу.
Аноним 22/11/17 Срд 14:26:15  361727
>>361710
Прям таки неизмеримую? То есть сингулярОЧКА может в Планковскую температуру? И зарождение своего персонального ПВК? (Пространственно Временной Континуум).
Аноним 22/11/17 Срд 15:38:06  361735
>>361727
Сингулярность нельзя рассматривать в контексте термодинамики совсем, поскольку она скрыта за горизонтом и что в ней делается доподлинно не известно. Но сама ЧД излучает, а раз излучает, то термодинамикой её описать можно. А если можно, то и вычислить температуру можно.
Аноним 22/11/17 Срд 15:58:11  361736
>>361676
>а на 0,51 c уже нет?
Видим. С хренов нет-то? На собственную скорость объекта насрать, хоть 99.99999 оно будет. К нам летит не сам объект, а свет от него, а скорость света всегда равна цэ. И не долетит свет только если удаление галактики больше цэ. Скорость объекта и скорость удаления со скоростью света это разные вещи, понимаешь? Если нас интересует видимость, скорость объекта можно выкинуть. Если он тусуется в пределах сферы видимости — ты его увидишь. Потому что даже если он летит на 0.99с вперёд, он всё равно будет посылать фотоны назад со скоростью 1с, а не 0.01с.
Аноним 22/11/17 Срд 16:22:46  361740
>>361736
Хм, действительно. То есть, получается, мы можем видеть объект, скорость которого относительно нас больше c (за счет прибавки из-за расширения пространства)?
Аноним 22/11/17 Срд 16:36:39  361744
>>361740
Да, если за счёт прибавки.
Аноним 22/11/17 Срд 17:03:48  361751
>>361744
Вроде понял, спасибо. Как в физике все сложно, пиздец просто.
Аноним 22/11/17 Срд 18:47:56  361753
>>361688
Гугли теорию инфляции, а петухов с их большим взрывом шли под шконарь смело, где им и место.
Аноним 22/11/17 Срд 18:52:15  361754
>>361753
Ничего что инфляция часть ТБВ?
Аноним 22/11/17 Срд 18:55:40  361755
>>361754
Ничего что БВ предполагает наличие первичной сингулярности?
Аноним 22/11/17 Срд 18:58:39  361756
>>361755
Как одно противоречит другому, космолог мамкин? Ты в курсе, как вообще появилась инфляционная модель? Это костыль ТБВ.
Аноним 22/11/17 Срд 19:20:07  361762
>>361756
Разумеется никак не противоречит. Ты прав, Анон.

Уебывай под свой шконарь, космолог хуев. Как одно противоречит другому вообще охуеть вопросы у него блядь.
Аноним 22/11/17 Срд 19:27:18  361763
>>361762
>In physical cosmology, cosmic inflation, cosmological inflation, or just inflation, is a theory of exponential expansion of space in the early universe. The inflationary epoch lasted from 10−36 seconds after the conjectured Big Bang singularity to sometime between 10−33 and 10−32 seconds after the singularity.
Ко-ко-ко, цитата из педивикии не пруф.
Аноним 22/11/17 Срд 19:36:59  361764
>>361762
Какой ты борзый петушок. Ты прилюдно обосрался, и еще визжишь? Какой молодец.
Аноним 22/11/17 Срд 19:40:51  361765
>>361762
- Модель говна лучше модели еды!
- Ничего что говно из еды получается?
- Врёти! Уёбывай!
Аноним 22/11/17 Срд 20:03:39  361767
>>361735
> А если можно, то и вычислить температуру можно.
только тепловой поток
Аноним 22/11/17 Срд 20:08:01  361768
https://news.rambler.ru/tech/38474049/?utm_content=rnews&utm_medium=read_more&utm_source=copylink

Так, посоны, у меня получилась цифра в 1280 кг/м^3 плотности. Что это такое? Кусок льда с вкраплениями металла и неметаллических соединений?
Аноним 22/11/17 Срд 20:08:50  361769
>>361768
> Кусок льда с вкраплениями
камней
Физон пошёл по пизде. Аноним 22/11/17 Срд 20:36:30  361774
image.png (34Кб, 416x402)
Аноны, есть одна хуйня недо-движок с недо-физоном, который я накалякал. И с этим движком есть проблема. Орбита единственного двигающегося объекта постоянно прокручивается, примерная траектория этого объекта изображена чёрной линией. И вот я задался вопросом: "Есть ли подобное ирл, или это погрешности вычислений?"
Я пошарился, и из всех известных мне игор с подобной фичей нашёл только одну - Spacewar, там орбита корабля тоже постоянно "прокручивается". Прошу вашей помощи с этим вопросом.
Аноним 22/11/17 Срд 21:09:31  361787
>>361774
ты случайно в ото
Аноним 22/11/17 Срд 21:10:16  361788
>>361774
меркурий
Аноним 22/11/17 Срд 21:12:20  361791
>>361787
Я случайно что?
Аноним 22/11/17 Срд 21:12:35  361792
>>361791
в ото
Аноним 22/11/17 Срд 21:12:37  361793
>>361788
Подумою
А с кометами похожая штука?
Аноним 22/11/17 Срд 21:13:47  361795
>>361793
у них больше реактивная тяга
Аноним 22/11/17 Срд 21:15:13  361796
>>361788
https://ru.wikipedia.org/wiki/Смещение_перигелия_Меркурия
Аноним 22/11/17 Срд 21:15:44  361797
>>361792
В ото?
Аноним 22/11/17 Срд 21:16:45  361798
image.png (69Кб, 673x306)
>>361797
чо тралеш?
Аноним 22/11/17 Срд 21:18:45  361799
image.png (46Кб, 383x479)
>>361798
Нет, просто пытался повторить историю с гранатой.
Аноним 22/11/17 Срд 21:20:57  361800
image.png (39Кб, 362x459)
>>361799
несмешно
Аноним 23/11/17 Чтв 13:48:37  361857
Такой вопрос: Можно ли послушать пульсары в домашних условиях?
Аноним 23/11/17 Чтв 13:52:15  361860
>>361857
Можно, но нужно иметь прямые руки, разбираться в радиотехнике и иметь место под тарелочку приличных размеров. Ну и денег полтора чемодана.
Аноним 23/11/17 Чтв 17:58:27  361899
>>361774
Да, ты обосрался с расчетами слегонца.
Чини физон, случайно ОТО не напишешь, надо специально искажения прописывать.
Аноним 23/11/17 Чтв 17:58:49  361900
Почему у Венеры нет спутника? Он мог быть раньше?
Аноним 23/11/17 Чтв 18:07:03  361901
>>361900
>Почему у Венеры нет спутника?
Аллах в своей мудрости не даровал.
>Он мог быть раньше?
Мог. Возможно даже это был Меркурий, но пока на этот счет никаких доказательств. Одни гипотезы.
Аноним 23/11/17 Чтв 18:22:58  361904
>>361767
Какой нахуй тепловой поток? Что ты несешь?
Аноним 23/11/17 Чтв 18:33:22  361906
>>361904
эквивалентную температуру абсолютно черного тела через светимость / тепловой поток вычислить можно
но это не сколь-либо реальная величина
Аноним 23/11/17 Чтв 18:34:24  361907
>>361334
Иисусе
Аноним 23/11/17 Чтв 18:50:45  361912
image.png (2Кб, 135x53)
>>361906
Температура - это в первую очередь термодинамика, а не градусник у кота в жопе, или свечение чего-то там.
Аноним 23/11/17 Чтв 19:34:37  361914
>>361899
Интересно, ибо я просто складывал вектор инерции и вектор гравитации. Уж не подскажете, что у меня не так?
Аноним 23/11/17 Чтв 19:40:37  361915
>>361914
К вектору скорости прибавляешь вектор ускорения, все должно быть окей, ЕМНИП. У меня получалось нормально лет десять назад.
Аноним 23/11/17 Чтв 19:56:20  361919
>>361914
Вектор тяготения на шаг времени помножить не забыл? Ну или, как вариант, у тебя такой конский шаг времени, что накапливающиеся ошибки видны невооруженным глазом.
Аноним 23/11/17 Чтв 19:58:37  361921
>>361919
Без умножения на тики был бы какой-никакой рандом. У него же четкая прецессия.
Может векторы не те использует, библиотечные какие-нибудь или что ещё.
Аноним 23/11/17 Чтв 20:24:29  361925
Спасибо, спейсаны. Подскажите где найти инфу о столкновении Космоса2251 и Иридиума33? Инфы вообще мало о причинах и так далее.
Аноним 23/11/17 Чтв 20:26:17  361926
>>361925
>о причинах
Всё банально. Они оказались в одно и то же время в одном и том же месте.
Щас Илон сотоварищи запустят пару тысяч спутников и триггернут синдром кесслера, так тогда эти столкновения ежедневно будут. Звездопады каждой ночью!
Аноним 23/11/17 Чтв 20:28:08  361927
>>361926
Оказаться в одном месте в одно время довольно сложно на таких пространствах. К тому же можно ли было избежать столкновения?
Аноним 23/11/17 Чтв 20:28:54  361928
2312.png (8Кб, 553x499)
Тело падает на Землю. На него действует ускорение свободного падения. Силой трения воздуха пренебрегаем. Падает тело в портал синий, а вылетает из жёлтого.
Как определить предельную скорость, которую достигнет это тело, ведь скорость света превысить оно не сможет, но на него действует ускорение?
Аноним 23/11/17 Чтв 20:33:53  361929
>>361927
Космос был в ауте, так что только иридиум пердануть мог (вроде у них есть движки коррекции) и запросто увернуться. Проблема в том, что следят в основном чтобы об МКС ничего не уебалось, на остальное посрать, шансы малы и стоит то барахло не полтораста миллиарда.
Аноним 23/11/17 Чтв 20:35:52  361930
>>361928
Просто прибавляй не к скорости, а к быстроте 10м/с2. Быстрота бесконечна.
https://ru.wikipedia.org/wiki/Быстрота
А когда захочешь узнать скорость, сделай преобразование.
Аноним 23/11/17 Чтв 20:36:28  361931
>>361929
Ну то есть всем было просто насрать, в том числе и Иридиуму? Ну уебется и хуи с ним? Их вообще НАСА предупреждало? Время равно же есть карта обломков, за ними по идее должны следить
Аноним 23/11/17 Чтв 20:43:41  361932
>>361931
Следят, конечно. Просто в этот раз система предсказания предсказала, что спутники разойдутся с приличным запасом.

http://celestrak.com/events/collision.asp

>
In the report issued on 2009 February 10 at 1502 UTC, SOCRATES predicted a close approach of 584 m between Iridium 33 and Cosmos 2251. This was not the top predicted close approach for that report or even the top predicted close approach for any of the Iridium satellites for the coming week. But, at the time of predicted close approach (1656 UTC), Iridium 33 suddenly went silent. The US Space Surveillance Network (SSN) subsequently reported that they were tracking debris clouds in both the Iridium 33 and Cosmos 2251 orbits, confirming a collision.
Аноним 23/11/17 Чтв 20:48:44  361935
>>361932
Ну вот, значит система сбой дала, а ты говоришь случайность. Хуй там.
Аноним 23/11/17 Чтв 20:49:15  361936
>>361932
Космос подрезал, кароч. Надо было оставаться на месте дорожно-транспортного космически-орбитального происшествия и ждать ГАИ Шаттл.
Аноним 23/11/17 Чтв 20:55:09  361937
>>361935
Тут ещё такой момент, что SOCRATES -- это система, которая пытается предсказывать по публичным неточным данным. После того случая STRATCOM начал сам рассчитывать предупреждения по более точной внутренней информации.

>New Procedures Since the Collision*
In June 2007, the U.S. military had been providing daily warnings to Iridium of potential collisions between its satellites and other objects. However, as a result of the sheer number of these warnings and the inaccuracy of the data provided by the U.S. military, these warnings were stopped at some point prior to the collision in February 2009.
After the collision, the U.S. military’s Joint Space Operations Center (JSpOC) added the entire Iridium constellation to its daily conjunction assessment procedures, using the same high accuracy data and calculations as are used to screen human spaceflight and high-value U.S. military satellites.
As of the end of 2009, the JSpOC had expanded these screenings to cover all of the almost 1,000 active satellites in Earth orbit. The JSpOC is currently providing warnings 72-hours in advance to the operators of all these active satellites of close approaches within 1 km for LEO and 5 km for Geostationary Earth Orbit (GEO).
U.S. Strategic Command (STRATCOM) has also put in place a new program called Space Situational Awareness (SSA) Sharing.11 This program is designed to share more information on the location and position of objects in orbit, as well as detailed analysis of that information, with program partners. This information is above and beyond the low accuracy Two-Line Elements that are currently made public by STRATCOM on the Space Track website (http://www.space-track.org).

https://swfound.org/media/6575/swf_iridium_cosmos_collision_fact_sheet_updated_2012.pdf
Аноним 23/11/17 Чтв 21:06:52  361938
>>361928
>Силой трения воздуха пренебрегаем
Сильно.
>Как определить предельную скорость
В таком случае, предельная скорость равна c. Она никогда не будет достигнута, но это предел.
Аноним 23/11/17 Чтв 21:09:23  361939
>>361938
А что будет останавливать? Ну, почему не сможет достичь?
Аноним 23/11/17 Чтв 21:13:45  361941
>>361939
Теорию относительности почитай. Ничего не будет останавливать, просто ускорение будет снижаться, оно только в школе постоянно.
Аноним 23/11/17 Чтв 21:16:13  361942
>>361941
И ускорение будет стремиться к нулю, но не достигнет его никогда, так?
Аноним 23/11/17 Чтв 21:18:54  361943
>>361942
Да. Рискую спиздеть, но вроде кинетическая энергия будет увеличиваться линейно по времени (пренебрегая трепыханием высоты), но для увеличения скорости надо будет все больше и больше энергии, поэтому ускорение будет падать и стремиться к нулю.
Аноним 23/11/17 Чтв 21:19:07  361944
>>361939
>>361942
При приближении к с одна и та же сила будет давать все меньшее ускорение. На релятивистских скоростях много ломающих восприятие обывателя эффектов возникает. Почитай википедию про это и послушай каких-нибудь Билл Найи ДеГрасс Тайсон Митио Каку Хокингов.
Аноним 23/11/17 Чтв 21:22:54  361945
>>361943
Да, спиздел. Линейно будет расти имульс. В ньютоновской механике он пропорционален скорости, так что скорость по-ньютону тоже должна расти линейно, но в ОТО (и реальности) это не так.
Аноним 23/11/17 Чтв 21:23:20  361946
>>361943
>>361944
>>361930
Спасибо.
Аноним 23/11/17 Чтв 21:28:06  361947
>>361945
Так ведь и кинетическая энергия должна будет расти тогда, нет?
Аноним 23/11/17 Чтв 21:34:42  361948
>>361947
Да, только нелинейно.
Аноним 23/11/17 Чтв 21:36:52  361950
>>361948
Почему нелинейно? При воздействии той же силы будет линейно и расти.
Аноним 23/11/17 Чтв 21:38:34  361952
>>361950
Потому что E = mv^2 / 2.
Аноним 23/11/17 Чтв 21:39:45  361953
>>361950
Кинетическая энергия при небольших скоростях равна mv^2/2. А вначале скорость растет практически линейно, значит энергия - квадратически.
Аноним 23/11/17 Чтв 21:40:59  361955
image.png (431Кб, 1600x900)
>>361952
>>361953
Всегда был троечником.
Аноним 23/11/17 Чтв 21:47:59  361957
Привет из сайфача. Вопрос: скажется ли на движении планет находящихся внутри орбиты плутона его внезапное исчезновение. Именно исчезновение: раз и нету, распылился.

И в общем: повлияет ли внезапное исчезновение небесного тела, на движение тел находящихся внутри его орбиты? А снаружи?
Аноним 23/11/17 Чтв 21:50:27  361958
>>361957
Плутон ОСНЕ мелкий. Он даже не планета. Всем похуй будет.
А вот жупитер и даже твойанус могут повлиять. Особенно жупитер. Правда, не катастрофически.
Вот заметное изменение эксцентриситета орбиты почти любого тела повлияет жопно.
Аноним 23/11/17 Чтв 21:51:28  361959
>>361957
Повлиет и на те, и на другие, но очень незначительно. Плутон так-то шибздик.
Аноним 23/11/17 Чтв 21:57:01  361961
>>361959
А в какую сторону? Внутренние тела сдвинутся к Солнцу?
Аноним 23/11/17 Чтв 21:59:14  361962
>>361961
>>361957
Поставь Universe Sandbox и попробуй сам. Базарю, понравится.
Аноним 23/11/17 Чтв 22:03:42  361963
>>361961
Нет, с хуя ли. Просто другие тела вносят некоторое "возмущение" в орбиты. Ну вот если нарисовать эллипс, в один его фокус запихнуть Солнце и пустить по нему планету, то в отсутствие других планет (и пренебрегая ОТО), она должна ровно по этому эллипсу бегать бесконечно. А за счет влияния других тел, эта орбита будет медленно поворачиваться. Типа сейчас у тебя вертикальный эллипс, а через сотню тысяч лет повернется до горизонтального. Ну, вот без Плутона не за 100000 лет, а за 100005, или 99995, где-то примерно вот так.
Аноним 23/11/17 Чтв 23:44:50  361980
>>361928
По идеи этот шар при увеличении скорости если это идеальное телепортирование, будет притягивать к себе землю, постепенно слетая её с орбиты. Но мне вот кажется, что 1. Портал будет делать гравитационную аномалию и нихуя не произойдёт. 2. если всё же аномалии не будет, то энергия всё равно должна от чего-то братся. И произойдёт какая-то неведомая хуйня. Или портал сломается или всё невелируется в 0.

Ещё когда в 1 портал играл, я уже тогда на интуитивном уровне понимал, что это всё такая дичь антинаучная. Мне сразу было не понятно почему скорость сохраняется?
Аноним 24/11/17 Птн 07:18:58  361995
>>361165 (OP)
1. Вселенная вечна или по крайней мере гораздо старше 13,7 млрд?
2. Черные дыры расщепляют материю на составляющие, а не ведут в другие вселенные?
3. темные промежутки между звездами это пыль?
4. темное вещество это обман типа: найди то, чего нет?
Аноним 24/11/17 Птн 07:36:29  361997
>>361995
До большого взрыва времени не было
Да
Водород
Временная затычка описания вселенной
Аноним 24/11/17 Птн 10:55:00  361999
>>361995
>(OP)
>1. Вселенная вечна или по крайней мере гораздо старше 13,7 млрд?
Нет.
>2. Черные дыры расщепляют материю на составляющие, а не ведут в другие вселенные?
Нет и нет
>3. темные промежутки между звездами это пыль?
Пыль и газ. В основном водород.
>4. темное вещество это обман типа: найди то, чего нет?
Это что-то, не понятно что, но существует. По всей видимости не может в электромагнитное взаимодействие, иначе бы давно нашли, зато в гравитационном весьма.
Аноним 24/11/17 Птн 11:34:00  362001
Вот смотрите: световая скорость и эффект Оберта. Берем ракету с идеальным химическим двиглом и постоянным УИ. Тогда при разгоне из-за эффекта оберта будет расти тяга. При достижении околосветовой скорости будет расти масса корабля, следовательно будет расти необходимая на дальнейшее изменение скорости энергия. Но из-за эффекта оберта будет расти тяга и следовательно прирост кинетической энергии. Получается, что нам будет нужно конечное количество топлива для достижения скорости света. Где я проебался?
Аноним 24/11/17 Птн 11:36:44  362002
>>362001
>нужно конечное количество топлива для достижения скорости света
Вот здесь.
Эффект Орбита не панацея, приведи формулу какой он прирост дает.
И вспомни, что энергия требуемая для одного и того же прироста скорости при релятивистской растёт экспоненциально.
Аноним 24/11/17 Птн 12:06:03  362004
>>362002
С хуя ли экспоненциально? E=mc^2, E=mv^2/2. Квадратично от скорости же
Аноним 24/11/17 Птн 12:09:10  362005
image.png (4Кб, 328x57)
>>362004
Экспоненциально.
>This formula shows that the work expended accelerating an object from rest approaches infinity as the velocity approaches the speed of light. Thus it is impossible to accelerate an object across this boundary.
Аноним 24/11/17 Птн 12:53:10  362006
>>362005
Вообще-то, это даже не экспоненциально, это еще суровее. Экспонента стремится к бесконечности когда аргумент туда сам стремится. А тут бесконечность уже при v → c.
Аноним 24/11/17 Птн 13:00:20  362007
>>362005
Хм, нихуя себе, однако. Как называется функция, где значение зависит от в том числе него самого, ну типа силы сопротивления воздуха?
Аноним 24/11/17 Птн 15:18:37  362021
>>361980
Я просто для наглядности пример с порталами выбрал, не бей.
Аноним 24/11/17 Птн 16:26:07  362026
Человечество может в будущем научиться нивелировать угрозу черных дыр и взрывов звездных систем?
Аноним 24/11/17 Птн 16:29:53  362027
>>362026
Для начала надо, чтобы эта угроза была.
Ну есть ЧД, ну бахают звезды, какая нам-то разница?
Если под джет попадем - пизда в любом случае без предупреждения. А солнце просто распухнет что твоя мамаша после родов и медленно поджарит нас.
Аноним 24/11/17 Птн 16:30:41  362028
>>362026
>угрозу черных дыр
Чем они тебе угрожают? Теоретическая возможность, что прилетит черная дыра и врежется в Солнце есть, практически вероятность этого наверное миллиардная доля процента за все время существования СС.
>и взрывов звездных систем
Взрывов сверхновых? Скорее всего, только улучшив методики предсказания. Это случается настолько редко, что постоянно экранироваться от гамма-вспышек немного накладно будет, даже если создать технологии для этого.
Аноним 24/11/17 Птн 16:55:49  362033
>>362026
Блжад, обычных звёзд, у которых есть хоть какой-то шанс столкнуться с Солнцем, в тысячи раз больше, чем ЧД, но люди почему-то спрашивают именно про ЧД. Чем ЧД так страшны-то? На более чем короткой дистанции от них они притягивают как обычные звёзды, звиздец начнётся только если ближе подлететь. Но на таком же расстоянии и звезда уже сжарит тебя, так что они равносильны.
Аноним 24/11/17 Птн 17:07:22  362034
>>362033
Черные дыры типа не видны, потому и боятся, что дескать вот живем себе, а потом - хуякс - съедены насмерть черной дырой, лол.
Аноним 24/11/17 Птн 20:23:47  362047
14887349585000.png (169Кб, 510x346)
>>362034
>звёзд в галактике 200млрд.
>ни одна не летит так, чтобы представлять опасность для СС
>бояться внезапной ЧД
Ладно, я понял общественную™ позицию по этому вопросу.
Аноним 25/11/17 Суб 02:35:30  362079
Матрица.jpg (37Кб, 400x400)
Во вселенной происходит куча дерьма, вроде взрыва сверхновых, гиперновых слияние всяких говен, и прочие яркие вспышки сопоставимые с яркостью галактики в которых онные находятся.
Какова вероятность произойти 2м и более таких событий произойти в одной и той же галактике одновременно т.е. свет до нас дошёл ~ в одно и тоже время, если учитывать что все галактики повёрнуты к нам строго перпендикулярно полюсами?
Аноним 25/11/17 Суб 03:39:14  362087
>>362079
Были бы достаточно мощные телескопы, можно было каждый день несколько регистрировать. Так-то в галактиках сверхновая в среднем по больнице ебашит раз в сто лет. Учитывая, что галактик в обозримой вселенной сотни миллиардов, то всегда можно было бы найти пару.
Аноним 25/11/17 Суб 04:20:51  362098
>>362047
>>362034
Ньюфаги не знают про Великий Аттрактор.
Мы не то, что всей галактикой, а всем галактическим кластером летим в него. И через N млрд лет жмякнемся в самую большую черную дыру. Так что вечности нет, бытие тщетно.
Но некоторые полагают, что в ВА вселенная схлопывается обратно, и когда масса станет достаточной, произойдет новый большой взрыв, и таким макаром бытие в каком-то смысле бесконечно.
Ну и какие выводы вы теперь можете сделать?
Аноним 25/11/17 Суб 05:57:49  362100
>>362098
Грустно стало. Обезяны конечно сдохнут к тому моменту и врядли станут медзвездным видом, но какая-то безысходность с этими говнами, которые будут пожирать материю.
Аноним 25/11/17 Суб 08:28:43  362103
>>362098
Атрактор это лишь центр притяжения.
Через н млрд. лет мы наоборот удалимся так как не удалимся мы только от местной группы. Остальные же улетают от нас.
Я честно не понимаю вас ребята.
Я в детстве тоже пугался ЧД. Но когда мне исполнилось 6 лет я уже понял беспочвенность данного страха. Чего взрослому человеку боятся я не понимаю.
Аноним 25/11/17 Суб 10:46:39  362107
>>362098
Ещё один петух с аттрактором
Вселенная расширяется быстрее, чем мы до него летим, еблан
Аноним 25/11/17 Суб 11:46:12  362108
>>362098
Для тобi тред есть https://2ch.hk/spc/res/329557.html >>329557 (OP)
Обсуждайте там свои беспочвенные страхи с единомышленниками, неча тут из пальца фобии высасывать.
Аноним # OP  25/11/17 Суб 13:16:25  362118
Тупой вопрос.
Кто сколько раз перекатывал этот тред?
Я вроде раза четыре.
Аноним 25/11/17 Суб 13:20:39  362121
>>362118
Я вроде джва раза перекатывал, один раз точно, на счет второго не уверен.
Аноним 25/11/17 Суб 13:44:46  362123
В СС есть объекты которые можно использовать для путешествий во времени? Солнце, Юпитер мб?
Аноним 25/11/17 Суб 14:02:54  362124
>>362123
Любой объект.
Сядь на табуретку и перемещайся в будущее со скоростью семь дней и пара наносекунд за неделю.
Аноним 25/11/17 Суб 14:10:33  362126
>>362124
Ты нагло врёшь, потому что на Солнце и газогигантов не получится поставить табуретку.
Аноним 25/11/17 Суб 14:47:42  362131
image.png (212Кб, 250x531)
>>362126
Табуретку из аэрогеля ставь на юпитер.
А Раката вообще табуретку на солнце ставили.
Аноним 25/11/17 Суб 15:42:52  362133
Как спускаемая капсула союза управляется в снижении? У ней же нет крыльев.
Аноним 25/11/17 Суб 16:31:18  362134
Космобляди, объясните мне вот это?

https://www.youtube.com/watch?v=Ibjw_MhDp8Y
Аноним 25/11/17 Суб 16:34:38  362135
>>362134
и вот это

https://www.youtube.com/watch?v=_dxzSv_96oM
Аноним 25/11/17 Суб 16:39:55  362136
>>362134
>>362135
Для тебя специальный загон есть на борде, мамкин тролль https://2ch.hk/zog/res/246013.html
Аноним 25/11/17 Суб 16:45:40  362137
>>362136
Я не троль и не зоговец. Я вполне адекватная шароблядь, просто поясни мне за видеоряд на первом ролике.

Нахя им юзать хромакей и делать каплю воды в фотошопе, а потом выдавать за real deal?
Аноним 25/11/17 Суб 16:47:19  362138
>>362137
Как насчет просто не смотреть зогоговно? При желании можно из хуя пальца высосать любую ахинею, найти "ляп" там где им и не пахло.
Видео не смотрел и не собираюсь.
25/11/17 Суб 16:48:45  362139
>>362133
Это называется спускаемый аппарат. У него есть двигатели системы управления спуском.
Аноним 25/11/17 Суб 16:49:22  362140
>>361980
Потенциальная энергия берется из телепортации
Аноним 25/11/17 Суб 16:53:09  362141
>>362138
То есть ты утверждаешь, что картинка космонавта на фоне левого задника - ЗОГомонтаж? По мне так ролик вполне реален. Мне похуй на то, что несёт автор и какие он там покровы срывает, просто, судя по многочисленным ляпам я вижу, что действительно имеет место НАСовский фотошоп в этих роликах. Вопрос только ЗАЧЕМ?
Аноним 25/11/17 Суб 16:59:27  362142
>>362141
>То есть ты утверждаешь, что картинка космонавта на фоне левого задника - ЗОГомонтаж?
>>362138
>Видео не смотрел и не собираюсь.
И ты не смотри.
Аноним 25/11/17 Суб 17:18:56  362144
13034043725365.jpg (32Кб, 330x296)
>>362142
Хорошо, забудем о видосах. Просто ответьте мне на утверждение. Наса юзает хромакей для создания роликов. Нахуя?
Аноним 25/11/17 Суб 17:20:58  362145
>>362144
Чтоб снять качественно, очевидно же.
Аноним 25/11/17 Суб 17:30:21  362146
Как-то это не очень правильно, не считаешь? Это же не боливуд. Тут как быпрелесть в натуральности.
Аноним 25/11/17 Суб 17:32:44  362147
1.jpg (106Кб, 989x777)
>>362146>>362144>>362134
Если я найду это видео, которое было на заднем плане во время визитов буша и докажу тебе, что это не хромокей, а просто экран для экспериментов, ты признаешь, что тебя возможно ввели в заблуждение плоскоземляне?
Аноним 25/11/17 Суб 17:38:13  362148
>>362147
Да я не против того, чтобы это был экран для экспериментов. Но какова цель этих экспериментов? Там же кусок каюты слева от занавески. и зеленый мячик на нитке, из которого будет сделана компьютерная вода, прямо как на уже смонтированном видосе.


Для своих экспериментов они могли бы и не извращаться подобными деталями, а делать всё спокойно на земле в другой одежде и в студии, а тут прямо видно, что они собираются всё это накладывать поверх картинки.
Аноним 25/11/17 Суб 17:39:58  362149
>>362148
Ты не ответил на вопрос. Если я найду это видео и точное описание эксперимента, который они пытались показать, то ты признаешь, что тебя вводили в заблуждение и возможно тебе не говорили всей правды, создавая иллюзию, что от тебя якобы что-то скрывают?
Аноним 25/11/17 Суб 17:40:32  362150
>>362144
Ёб твою мать, это не хромакей, а сетка для автоматического отслеживания движущихся объектов. Ты вообще в курсе, почему хромакей однотонного цвета, а не измазан контрастной хуитой?
https://www.youtube.com/watch?v=fJgL3Rxsf3c
Аноним 25/11/17 Суб 17:46:27  362151
>>362149
Конечно признаю. Я здесь для поиска истины в первую очередь.
Аноним 25/11/17 Суб 17:52:20  362154
1.jpg (512Кб, 1280x1024)
>>362151
Хорошо, я рад это слышать.
Вот видео, где астронавт крутит шарик над этим экраном
https://www.youtube.com/watch?v=Eu6N6a9YErA
и мы видим, что они используют экран для того, чтобы камера могла создавать точки на каком-то графике. Ясно, что их камера не может создавать эти точки позиции, если фон будет другим.

Вот конкретно то видео без всяких точек
https://www.youtube.com/watch?v=ekgTjOEBdsY
Здесь они крутят шарик в горизонтальной плоскости.
Вот видео, где они опять используют экран и оборудование камеры в эксперименте для создания точек на экране
https://www.youtube.com/watch?v=mozXv8QQhes

зачем это делается? Для учебного пособия в школах в демонстрациях экспериментов по физике кругового движения
https://astroacademy.org.uk/resources/circular-motion/
На этом сайте можно скачать все эти видео, документы и материалы.

Смысл в том, что все это уже давно известно, но плоскоземляне не спешат удалять свои видео, в которых они говорили что якобы это секретное видео перед хромокеем, потому что они знают, что их аудитория не станет проводить расследование, сама искать эти видео экспериментов и их описание, они знают, что аудитория анти-космических видео верит им на слово.
Аноним 25/11/17 Суб 17:53:53  362155
>>362151
А вот подробный гайд для учителей, как учить физике на основе этих видео
https://astroacademy.org.uk/wp-content/uploads/2016/10/6.2491_Astro-Academy_Circular-Motion_AW.pdf
Аноним 25/11/17 Суб 18:00:32  362156
>>362154
О как. Это ставит всё на свои места. Спасибо, анон.

Кстати в плоскотреде меня уже успели на хуях оттоскать
Аноним 25/11/17 Суб 18:53:32  362159
>>362156
Нахуй, ты туда заходишь? Плоскоземелье - это массовый троллинг.
Аноним 25/11/17 Суб 19:44:49  362169
>>362156
> плоскотреде
Лол, где такой? Кинь ссыль, будь добр.
Аноним 25/11/17 Суб 20:00:11  362170
>>362169
>>362136
Троллеркостер уже 18 тредов
Аноним 25/11/17 Суб 20:01:04  362172
Cgange5t1.jpg (309Кб, 1280x781)
5e.jpg (2473Кб, 3500x2563)
dde3a753121f.jpg (51Кб, 600x513)
>>362133
https://youtu.be/-l7MM9yoxII?t=12m19s

>Управле­ние спуском осуществлялось с помощью газовых реактивных двигателей (ГРД), работаю­щих на перекиси водорода. В СУС система управления спуском) входили два ГРД по рысканию и два ГРД по тангажу тягой по 7.5 кгс, а также два ГРД по крену тягой по 15 кгс
Аноним 25/11/17 Суб 20:28:47  362173
>>362172
Че эти гигантские китайцы делают с Союзом!?
Почему нет по тангажу?
Аноним 25/11/17 Суб 22:08:52  362177
>>362170
> Троллеркостер уже 18 тредов
Не ну это конечно перебор, все читать не буду. Скрольну последний.
Аноним 25/11/17 Суб 22:11:06  362178
>>362172
Блэт, три дня назад в музее Алабина в Самаре был, там на входе эта хуевина стоит, тоже обгоревшая. Сзади типо не большого отсека такого пустого было, думаю там был парашют, так ли это?
Аноним 25/11/17 Суб 22:36:17  362181
6048740-2730300.jpg (60Кб, 423x500)
>>362178
да ето так
Аноним 25/11/17 Суб 22:51:54  362185
>>362181
Да именно про это я и говорил, значит правильно подумал. Добра.
Аноним 26/11/17 Вск 01:42:56  362204
Представь себе что человечеству человек чужда экспансия. Человечество занимало бы новые территории только когда к этому вынуждало перенаселение. Мир в котором человечество в 2017 состоит из 3 млрд живущих в основном в Африке в теплом климате в уютных городках... Остальные континенты ещё предстоит заселить.

Никаких проблем с экологией, энегретикой, войнами и прочей современной фигней. Да, технологии 18 века и в космос ещё никто не летал.

Но как лампово-то!

Ну вот почему нам так надо все занять и везде лезть, отталкиваясь других локтями?
Аноним 26/11/17 Вск 01:49:28  362205
>>362173
в смысле нет? А по центру одиночный не он? Один с одной стороны, другой с обртаной. Они и на фото есть
Аноним 26/11/17 Вск 01:49:53  362206
>>362204
Не хочу быть нигрой.
Аноним 26/11/17 Вск 02:24:05  362207
>>361165 (OP)
Первое: можно ли в космос на дирижабле вместо ракеты? Они, вроде, могут подниматься до нижней границы мезосферы, а там уже чуть ли не технический вакуум. Ставим на наш шарик ионный двигатель и включаем вот там, наверху. Получится ли разогнаться и взлететь выше?
Второе: читал я на википедии про вот это:
https://en.wikipedia.org/wiki/High-altitude_platform_station
Какие там могут быть технические проблемы? Что там так долго разрабатывать? Почему они еще не заменили спутники связи?
Третье: почему космос прокачивают так медленно, где альтернативные способы запуска? Есть же, кроме космического лифта, еще космические башни, например, которые могут работать как платформы для вот этого:
https://en.wikipedia.org/wiki/Space-based_solar_power
Аноним 26/11/17 Вск 02:34:07  362208
uASCatufUao.jpg (152Кб, 960x960)
>>362204
>Представь
Представил.
>Человечество занимало бы новые территории только когда к этому вынуждало перенаселение.
Ресурсы заканчиваются быстрее, чем свободное место. Поэтому и миграции, и экспансии. А если аноним с сосача запретил мигрировать, пока не встанет проблема перенаселения, то будет намного проще просто перебить лишние рты, конкурирующие с тобой за место и прочие ресурсы и дальше нихуя не делать.
>Мир в котором человечество в 2017 состоит из 3 млрд живущих в основном в Африке в теплом климате
Потомки тех, кто забил на экспансию и остался в Африке, в большинстве своём верят в волшебные мази от пуль и что фотография отнимает душу, зато не верят в ВИЧ и нихуя не делают благодаря гуманитарной помощи от сердобольных потомков тех, кто уебал в холодные ебеня некоторое количество сотен тысяч лет назад в поисках лучшей доли и начал понемногу развиваться, чтобы не помереть. Сидение на месте = деградация.
>в уютных городках.
Ты городки тамошние видел вообще? И я не про те, что колониального периода и построены европейцами.
>Никаких проблем с экологией, энегретикой...и прочей современной фигней.
Ну так уёбывай куда-нибудь в незаселённые территории, я лично предпочитаю отравленный выхлопами город, где у меня есть тёплая квартира, магазин и интернет, а вероятность быть сьеденным и высранным каким-нибудь мимольвом или мимомедведем крайне мала.
>войнами
Ну так уёбывай обратно в свою хиппи-коммуну, если не знаешь, что успешно и остервенело воюют не только поклонники ортодоксального племенного строя, и даже не только разные стаи обезьян, но и стаи волков и прочей социализированной негуманоидной живности. Deal with it.
>технологии 18 века
Откуда им взяться, если ты живёшь сотни лет без изменения окружающих условий и все твои проблемы решаются копьём и луком? Спроси у Номоле, Сентинельцев и прочих голожопых противников экспансии, которые за СРАНЫХ 50 ТЫСЯЧ ЛЕТ нихуя кроме этого самого копья и лука не придумали.
>Но как лампово-то!
Дело вкуса, знаешь ли. Мне к примеру вполне лампово, когда мои яйца спокойно лежат в трусах, а не колышутся на горячем ветру саванны или собирают ядовитых насекомых с листьев влажных джунглей.
>Ну вот почему нам так надо все занять и везде лезть, отталкиваясь других локтями?
Потому что мы не овощи, а ещё потому что жизнь не знает жалости, сучка - выживает сильнейший, выживает умнейший, а в любую секунду по нам может пиздануть случайный джет и мы все умрём, даже не успев понять, что это было. Такова Вселенная, как бы это не ужасало твою помутнённую избытком эстрогена центральную нервную систему.
И вообще, что ты за нюни тут развёл, у нас раздел про космос вообще-то.
Аноним 26/11/17 Вск 05:55:03  362212
>>362208
>не верят в ВИЧ
Зато верят в СПИД - и в то, что от него можно вылечиться, выебав девственницу. А если ещё съесть при этом нигру-альбиноску, то вообще станет полностью охуенно.
мимо любитель этнографии
Аноним 26/11/17 Вск 06:01:39  362213
>>362204
>чужда экспансия
Они из Африки полезли на север (и далее везде) не из каких-то абстрактных соображений, типа экспансии ради экспансии, а просто в поисках чего бы пожрать. Поинтересуйся, какая территория угодий нужна охотнику-собирателю / семье таковых. Рост населения --> нехватка жратвы --> молодежь уходит на поиски новых мест.
Аноним 26/11/17 Вск 06:44:19  362214
15043825125620.png (1515Кб, 1600x1600)
кто и как определи, что андромеда летит навстречу моечному пути?
может, он наоборот отдаляется, просто пока она отдаляется центром и вращается, в направлении нас, наблюдателей, приближается пик максимального приближения одного из рукавов, что накладывается на отдаление нашего рукава?
Аноним 26/11/17 Вск 07:08:24  362215
>>362214
Там не идиоты эти циферки считают. Вращение бы учитывали.
Аноним 26/11/17 Вск 07:11:58  362216
>>362208
>а в любую секунду по нам может пиздануть случайный джет и мы все умрём, даже не успев понять, что это было
Чет не верю, что прям любой случайный джет прямо вот так сразу нахуй за одну наносекунду, этож какая мощность должна быть?! Это получается спустя считанные секунды от такого плотного потока радиации, даже в среднем по планете начнёт повышаться на десятки Кельвинов. Но всё равно не ужели прямо МГНОВЕННО, несколько десятков секунд то наверное ещё будет возможность по пялится на ебовейшее запекание всего и вся, прежде чем ткани составляющие твои органы начнут фатально распадаться до древних основ.

Можно по подробнее про случайный джет = пизда вообще сразу? Просто я считал что мгновенно пизда и ничего понять не успеем может быть только при распаде ложного вакуума или ВНЕЗАПНОГО большого разрыва\ фатальных изменений физических постоянных и т.п.
Аноним 26/11/17 Вск 07:14:39  362217
>>362216
Забыл ещё СТРАННЫЕ КВАРКИ, правда не факт, что они прямо мгновенно бы всё преобразовали.
Аноним 26/11/17 Вск 07:25:30  362218
>>362216
>несколько десятков секунд то наверное ещё будет возможность по пялится на ебовейшее запекание всего и вся
>будет возможность
Толку-то.
Аноним 26/11/17 Вск 07:26:54  362219
>>362215
Ага, не идиоты.
ВСЯ современная наука, занимающаяся исследованием космоса за пределами СС - гадание по осциллографу и проверка тоненьких линий спектрометра. Показания спектрометра - это и есть факты. Всё остальное анал-карнавал на принципе "ехал наверное через вероятно, да скорее всего погоняя". 98% современных знаний о том, чё там у хохловзвёзд и космических тел где-то за незримой нематериальной границей СС - не факты, а предположения, причём голословные и существующий только потому, что никто не предлагает других.
Аноним 26/11/17 Вск 07:27:25  362220
Кстати гипотетически на какое расстояние надо отвалить от "точки старта", что бы оказаться вне досягаемости - случайного джета?

>>362218
>Толку-то.
Ну это да... Но таки понять успеем что ВСЁ придёт вот прям ща совсем скоро..
Аноним 26/11/17 Вск 07:31:03  362221
>>362219
>Всё остальное анал-карнавал на принципе "ехал наверное через вероятно, да скорее всего погоняя"
Пока другого нет приходится жрать то что технологии позволяют, хоть так что то знаем, конечно было бы в 100500 миллионов раз охуеннее если бы в хотя бы самые интересные места нашей галактики, можно было бы понапихать АМС и Роверов ещё бы мобильных телескопов не помешало, но возможностей нет ибо потенциальный барьер требуемых энергетических и временных ресурсов для такого исследования катастрофически велик для известных нам возможностей.
Аноним 26/11/17 Вск 09:59:45  362223
>>362205
Я долблюсь в глаза, спасибо.
Аноним 26/11/17 Вск 10:33:02  362224
>>362219
Вращение Андромеды учтено. Одна ее часть приближается у нас чуть быстрее чем другая, вращающаяся от нас вокруг центра Андромеды
Аноним 26/11/17 Вск 10:34:44  362225
>>362204
>экспансия
Это биологический императив, простой адаптационный механизм.
Кончилась жрачка в плейснейм - пошли в другое место и попробовали жрать там, адаптировались етц. Так ВСЕ животные действуют, мы не исключение.
Аноним 26/11/17 Вск 10:36:48  362226
>>362207
Ты только что взлёт на шариках из омского космодрома, только усложнил.
Дирижомбели поднимаются низенько, там как минимум четверть атмосферы еще есть.
Ионник будет немножко пукать и давать ничтожно малую долю тяги. Даже если представить, что он всплыл почти за атмосферу и плавает там, то ионником он не оттолкнется настолько, чтобы преодолеть гравитацию. Так и будет плавать в атмосфере, чуть быстрее, чем без ионника.
Аноним 26/11/17 Вск 10:38:49  362227
Есть предел скорости света. А почему именно этот предел, то есть почему именно это число? И ещё один тупой вопрос: связаны ли между собой фундаментальные константы?
Аноним 26/11/17 Вск 10:49:59  362228
>>362227
> А почему именно этот предел, то есть почему именно это число?
Так получилось...
Ещё вариант: было бы другое число, нас бы тут не было (вероятность 99.999999999999999%) или был бы кто то другой (вероятность 0.000000000000001%) Оценки от балды и могут сильно отличаться от разных точек зрения
Аноним 26/11/17 Вск 10:54:16  362229
>>362224
Это обычный оптический эффект. Если взять огромную монетку, подключить её центром к оси вращения, вырезать из монетки округлую свастику и посадить тебя на стул перед ребром так, чтобы твоё лицо находилось на близком расстоянии, для тебя, наблюдателя, находящегося по ребру с одной стороны монетки, тот рукав свастики, который ближе к тебе, будет выглядеть более быстро вращающимся, чем противоположный задний. Хотя на самом деле все рукава свастики вращаются с одинаковой скоростью.
Аноним 26/11/17 Вск 12:08:14  362231
>>362216
Джет это сильная гамма в первую очередь. Обращенную к джету половину планеты стерилизует моментально и вскипятит океаны и выпарит атмосферу. Другая половина задохнется в результирующем апокалипсисе.
Но джеты хоть и люто мощные, есть квадрат расстояния, так что бояться надо только ближних сверхновых, да еще и они узконаправлены, так что шансы ничтожно, астрономически малы.
Аноним 26/11/17 Вск 12:11:13  362232
>>362217
А они разве не случаются только в кварковых звездах из-за энергетического потенциала чтобы преодолет принцип Паули и занимать меньше места? "uud" кварки займут больше места, чем "sud"
Аноним 26/11/17 Вск 13:14:25  362236
>>362231
Какой толщины эти опасные джеты могут быть? То есть например свалившие на Марс смогут не поджариться? Или надо на миллиард км минимум от Земли? (не по курсу Джета естественно) может он настолько широкий, что уцелеть можно будет только свалив в соседнюю звёздную систему? Дальше чем в соседнюю?
Аноним 26/11/17 Вск 13:16:42  362237
>>362236
Слышал что можно задетектить Гамма всплеск по резко увеличивающейся плотности потока Нейтрино, так как те образуются раньше при сверхновой. В голову лезет сифятина в стиле "Спрячем хоть кого то на время всплеска за газовый гигант\звезда нейм"
Аноним 26/11/17 Вск 13:39:33  362238
Что скажете про такие 2 курса?

https://stepik.org/course/650/syllabus
https://stepik.org/course/2119
Аноним 26/11/17 Вск 14:21:27  362242
>>362231
>джеты
>сверхновые
Разве джеты это не потоки говн на релятивистских скоростях, которые запускают центры некоторых голактик? Причем тут сверхновые?
Аноним 26/11/17 Вск 14:39:48  362243
>>362242
Джеты с полюсов это не только ценный мех поток частиц, но и почти весь спектр ЭМИ, больше гаммой. Как раз благодаря тому, что на полюсах эти частицы разгоняются до реляскоростей и раскалены так, что фонят.
Аноним 26/11/17 Вск 15:08:36  362247
>>362243
Но запускают-то их смчд, а не сверхновые?
Аноним 26/11/17 Вск 16:14:23  362250
>>362247
Джеты бывают и при коллапсе сверхновой, когда куча материи падает на результирующую ЧД или нейтронную звезду.
Аноним 26/11/17 Вск 17:04:32  362256
Red-Bull-Strato[...].jpg (127Кб, 1000x898)
41.jpg (24Кб, 420x324)
image.png (366Кб, 724x744)
>>362207
1) дирижабли выше 20 километров никогда не поднимались, экспериментальный от локхидов для 18 километров и тот не взлетел взлетел до 9 километров и разбился

2) Рекорд высоты для аэростатов - 53 километров, выше ни один аппарат не поднимался

3) Всё это летает за счет силы архимеда, нету воздушной массы - нету выталкивающей силы. В безвоздушном пространстве твои дирижабли не могут летать по определению.

4) Чтобы поднять макаку-австрийца на 40 километров + 1700 килограмм обшивки и капсулу в 1300 кг пришлось городить пикрил.



А дальше смотри:

40 км, -30C. Соотношение молярных масс газов одинаковое: 4 к 29. Давление тоже. Тогда куб.м. гелия имеет массу - ~0.5 грамм, воздуха ~ 4 грамма. 849000+ кубов баллона дают 3 тонны+ подъемной силы. Похоже на реальный результат.

Теперь для 60 километров:
Температура та же. Куб воздуха весит 0.032 грамма, куб гелия - 0.0046. Соответственно положительная плавучесть с 1 куба - 0.0274 грамм. Чтобы поднять те же 3 тонны (обшивка, которая в реальности будет на несколько порядков весить больше + полезный вес в 1400 кг) нужно 109 000 000 кубов газа. И шар исполинских размеров (с исполинской массой), который в себя это поместит.

Скорее всего нету такого материала на Земле, который смог бы поднять хотя бы 3 тонны на 60 километров. Плюс, не забывай про управляемость такой йобы и безопасность. А ты еще предлагаешь с неё ракеты запускать, лол. Я даже не говорю про то, что запуск со 100 километровой высоты каких-то кардинально меняющих результатов не даст. Да, пропадут потери на сопротивлении воздуха, частично пропадут за счет меньшего времени до выхода на орбиту гравитационные и чуть меньшая скорость нужная будет до первой космической. Но это несколько десятков процентов, не более.
Аноним 26/11/17 Вск 17:08:33  362257
Вообщем милости прошу в омского кбстроения-тред, там такое любят
Аноним 26/11/17 Вск 17:09:09  362259
>>362256
>несколько десятков процентов
Полтора-два десятка.
Подавляющая часть дельты идет на разгон.
Из 10к дельты 1.5-2 км/с уходит на атмосферные и гравитационные потери, оставшиеся 7.8км/с - орбитальная скорость.
Аноним 26/11/17 Вск 17:37:42  362261
>>362256
Хотя получается шар всего в 300 метров радиусом. Насчет нескольких порядков разницы по массе я неправ, как и насчет материалов. Запилить можно, и посчитать все можно, но мне уже лениво.
Аноним 26/11/17 Вск 17:39:12  362262
>>362261
>>362256
А чому ты гелий считаешь? Водород же легче. Мы же не паксов через атлантику возить собираемся.
Аноним 26/11/17 Вск 17:40:14  362263
>>362261
Криво написал, это для той же массы шара 300 метров. Для реальной массы шар будет еще больше
Аноним 26/11/17 Вск 17:43:18  362264
>>362262
Водород не сильно проигрывает гелию (2 к 29 против 4 к 29), но токсичен и горюч. Но его нахер
Аноним 26/11/17 Вск 17:44:25  362265
>>362264
не сильно выигрывает /fix

Все, перезанимался.
Аноним 26/11/17 Вск 17:45:56  362266
fukken lold8.jpg (83Кб, 682x600)
>>362264
>водород
>токсичен
А про горючесть я уже написал - мы не паксов возить в америку собираемся, у нас уже и так ракета которая по статистике опаснее даже водородных дирижаблей.
Аноним 26/11/17 Вск 18:07:03  362268
>>362256
>взлетел
>до 9 километров
>и разбился
Об небесную твердь.
Аноним 26/11/17 Вск 19:41:22  362271
>>361332
хмм, закрыть дырявой крышкой и влить рюмасик через дырку
Аноним 26/11/17 Вск 19:44:58  362272
>>362268
Шутки шутками, но о небесную твердь можно разбиться. Вот когда протон при том пресловутом старте повернул на 90 градусов аэродинамическая нагрузка его сломала нахер.

>>362271
Зачем рюмасик переводить?
Есть же кипяток. Или средство для прочистки труб шоб наверняка.
Аноним 26/11/17 Вск 20:56:31  362281
2MiFYHENVuE.jpg (91Кб, 1200x826)
>>362266
Ещё как токсичен! От дигидрогена монооксида знаешь, как люди отравляются? Штабелями! И все до одного - с летальным исходом.
Аноним 26/11/17 Вск 21:45:27  362283
Говорят, что черные дыры не излучают свет. А какие они на вкус?
Аноним 26/11/17 Вск 21:46:45  362284
>>362283
На вкус как ампутация языка.
Она уничтожит вкусовые рецепторы.
Аноним 26/11/17 Вск 21:46:49  362285
>>362283
Как земля
Аноним 26/11/17 Вск 22:50:39  362292
>>362204
>технологии 18 века.
Вся эта подростковая романтика заканчивается ровно в тот момент, когда батя наступил ногой на острый камень и здох от гангрены. Жена здохла при родах. Половина детей, которую она успела нарожать до этого изуродована полиомиелитом. Мамка здохла от воспаления легких. Один Антоша Сычев сидит и надеется, что вот этот понос, который его донимает третий день не дезентерия, а просто отравление.
Аноним 26/11/17 Вск 23:24:35  362294
>>362283
Уже второй раз за полгода в ТТВ этот вопрос. Наверное, это капчуют младенцы, у которых кусать/сосать/пробовать всё на вкус - один из основных инструментов познания окружающего мира. Нихуя себе младенцы грамотно пишут, кстати.
Аноним 27/11/17 Пнд 00:22:40  362297
>>362219
>гадание
Погрешности рассчитываются же, как вероятностные, так и вызванные несовершенством приборов, ограничениями вычислительных машин.
>>362226
Так мы же его не против силы тяжести направляем, чтобы обнулить вес, а пытаемся вывести на орбиту, то есть разогнать до первой космической. Сила тяжести компенсируется силой Архимеда. Вопрос в том, достаточно ли силы Архимеда, чтобы поднять девайс на такую высоту, где трение об атмосферу не будет мешать разгону ионным двигателем до первой космической.
>>362256
>нужно 109 000 000 кубов газа
Нет же, где то в 10 раз меньше, ведь у тебя изменилось только давление в 10 раз, значит масса куба тоже уменьшилась в 10 раз. Но да, шар получается метров 200 в диаметре.
Есть еще футуристический хайтек вариант сделать оболочку шара из плазмы, а энергию сообщать микроволновкой прямо с земли. Вон, такую йобу уже патентуют:
https://www.sciencealert.com/boeing-has-patented-a-plasma-force-field-to-protect-against-shock-waves
Правда, тут беда с тем, что шарик должен будет сделать несколько оборотов вокруг Земли, прежде чем разгонится - нужны станции слежения за этой штукой во многих точках Земли. Они, правда, уже есть - радары дальнего обнаружения со времен холодной войны.
Аноним 27/11/17 Пнд 00:25:34  362298
>>362297
>патентуют
Чистая наука!
Аноним 27/11/17 Пнд 00:45:21  362300
БОМБА
О
М
Б
А


Посоны, в теории, этот торрент - бомба!!! https://rutracker.net/forum/viewtopic.php?t=5484910
Аноним 27/11/17 Пнд 01:05:12  362301
AS12-50-7326.jpg (1535Кб, 3000x3000)
>>362300
Да ето так.
Это всё взято отсюда, если не хочешь загружать все пикчи подряд:
https://www.flickr.com/photos/projectapolloarchive/
Аноним 27/11/17 Пнд 01:07:09  362302
>>362301
В том и дело, что лучше разом качнуть 50 гигов и наслаждаться в АЦДЦее.

Фантастическое совпадение, я как раз смотрел видосы про миссии Аполлон и мне пришло в голову поискать фотачки на Рутрахере, и как раз вчерась выложили!
Аноним 27/11/17 Пнд 01:08:10  362303
>>362301
И что там на фотке за кусок летит?
Аноним 27/11/17 Пнд 01:14:34  362304
1968.10.22YNVZu[...].jpg (726Кб, 2424x2424)
>>362302
>лучше разом качнуть 50 гигов
Там много почти одинаковых или не очень интересных - как по мне, лучше пересматривать и схоронять самые годные. Ну, дело вкуса.
>>362303
Один из лепестков обтекателя третьей ступени.
Аноним 27/11/17 Пнд 06:48:59  362309
>>362294
Ну вообще химики и в 19 веке пробовали вещества на вкус. Отчего, как думаешь, кислоты получили такое наименование? Нет, не из-за кислорода в составе. Их таки дегустировали, пусть и в растворенном виде. Кислород был так назван, потому что теория была дескать кислоты без кислорода в составе быть не может, в последствии, как оказалось, ошибочная.
Так что шутка про лизание камней в андромедном лишнем стволе не совсем шутка, а вполне рабочий метод научного познания.
Аноним 27/11/17 Пнд 07:35:19  362311
>>362309
>а вполне рабочий метод научного познания
>познания
Ну и что там познаётся кроме названия? Учитывая, что мы уже выяснили: определять наличие кислорода по вкусу — способ не очень.
Аноним 27/11/17 Пнд 08:24:43  362314
>>362309
Я тоже кислоты дегустировал, лол. Теперь я учёный?
Пробовал серную в трёхпроцентном растворе, кислая пиздец
Аноним 27/11/17 Пнд 09:49:32  362316
15066828727471.png (266Кб, 500x375)
Анон, хочу увидеть небо всех цветов. Вот у нас небо голубое. В обитаемой области жёлтой звезды у нас голубое небо. А если будет красная звуезда, небо будет зелёное?
А при голубой звезде - фиолетовое?
Назовите мне условия, при которых можно будет увидеть на поверхности планеты и бежевое небо, и пеочного цвета, и орнажевое, как персик, и розовое, и пурпурное, и зелёное, лимонное, и белое и синее синевы.
Аноним 27/11/17 Пнд 10:18:11  362320
>>362297
>Вопрос в том, достаточно ли силы Архимеда, чтобы поднять девайс на такую высоту, где трение об атмосферу не будет мешать разгону ионным двигателем до первой космической.
Нет.
ГОЧЕ летал ионником на низкой используя крылышки.
Попробуй приделать своему дирижаблю крылышки.
И да, го в омское КБ, там тебе расчеты проведут.
Аноним 27/11/17 Пнд 10:56:45  362321
>>361165 (OP)
Начпис - твёрдый-фантаст вкатывается в тред.

Если взять звезду спектрального класса O, со светимостью, например, в миллион солнечных, и укомплектовать её продвинутой астроинженерией, телепортирующей в ядро водород, то мощность на квадратный метр поверхности планеты, как на земле, будет примерно на расстоянии 1000 а.е., так? (У меня в такой системе Предтечи жить будут).

Нужно-ли какие-то поправки на спектр брать? Повлияет ли то, что большая часть излучения будет ультрафиолетом на положение обитаемой зоны вокруг звезды?
Аноним 27/11/17 Пнд 11:39:28  362323
Вот такой вопрос. Вот есть у нас детектор грав волн 2,5 в относительно стационарном положении. А вот хотят сделать похожую штуку в космосе. Как они собираются учитывать неизвестные, невидимые астероиды, которые притягивая спутник, будут искажать результаты похлеще, чем какое либо возмущение на земле?
Аноним 27/11/17 Пнд 12:23:27  362334
>>362321
>твердый-фантаст
>телепортировать водород в ядро
Аноним 27/11/17 Пнд 12:33:57  362336
>>362334
ОТО червоточины не запрещает.

Водород в ядро - чтобы звезда долго жила.
Аноним 27/11/17 Пнд 12:43:50  362337
>>362336
У тебя эта червоточина ядро выплевывать будет в то место, откуда ты водород собрался телепортировать.
Или ты в своей твердой фантастике придумал односторонние червоточины?
Аноним 27/11/17 Пнд 12:52:13  362339
>>362337
Односторонняя не нужна, я эту тему продумал.

Есть более старая звезда O-класса, в её ядре парциальное давление водорода ниже, а гелия - выше, на орбите вокруг неё крутится планета предтеч.

Есть более молодая звезда того же O-класса, на орбите которой никого нет.

Давление и температура в ядрах примерно одинаковая.

Соединяем ядро первой с ядром второй червоточиной. Из-за разности парциальных давлений гелий будет перетекать с первой на вторую, а водород - с второй на первую.

Но меня сейчас интересует спектр и влияние на климат.

Киловатт на квадратный метр видимым и инфракрасным (Солнце) будет в плане влияния на климат отличаться от киловатта на метр квадратный видимым и ультрафиолетом (O-класс)?
Аноним 27/11/17 Пнд 12:52:19  362340
>>362323
У них разные сигнатуры. Мимоастероид будет мягко нарастать и потом так же убывать.
Гравиволны от сливающихся ЧД это с нарастающей частотой и амплитудой колебания и резкий выброс гравиволн. Не спутаешь.
Аноним 27/11/17 Пнд 12:54:53  362342
image.png (712Кб, 877x488)
>>362339
>Киловатт на квадратный метр видимым и инфракрасным (Солнце) будет в плане влияния на климат отличаться от киловатта на метр квадратный видимым и ультрафиолетом (O-класс)?
Будет. Я не иксперд нихуя и точно не скажу, но факт есть в том, что многие молекулы (тот же полиэтилен) распадаются под УФ, всякие озоны тоже иначе существуют. Изменение спектра повлияет на экологию, но как конкретно - надо много шерстить хотя бы ту же википедию. Я думаю, тебя пошлют здесь в нужном направлении.
Аноним 27/11/17 Пнд 13:00:38  362343
>>362342
Я предполагаю, что жёсткий УФ до поверхности планеты не долетит, земная атмосфера вообще непрозрачна для него.

Пытался нагуглить готовые модели типа MODTRAN (модель пропускания света земной атмосферой), но они, по очевидным причинам, построены для видимого диапазона и ближних ИК и УФ.

Атмосферу, кстати, планирую с большей концентрацией кислорода, чем земную.
Аноним 27/11/17 Пнд 13:04:52  362344
>>362343
Жесткий УФ хоть и не долетит до поверхности, верхнюю атмосферу жарить будет будь здоров.
Аноним 27/11/17 Пнд 13:12:13  362345
>>362344
А вообще - у меня такая экзотика потому, что Предтечи так прячутся от более молодых рас.

Никто, во-первых не предполагает жизни около O-звезды, во-вторых её излучение замечательно скрывает избыток ИК от техники.

Спалить можно только по аномальному сроку жизни такой звезды, но младшие расы недостаточно долго ведут наблюдения для этого.

Ещё думаю о кислород-гелиевой атмосфере, но сомневаюсь в твёрдости. Идея такова:
1) Интенсивный звёздный ветер, состоящий в основном из ионизированного водорода и гелия разлетается по всей системе (а излучение его ещё и подгоняет);
2) Частицы оного ветра захватываются магнитным полем планеты и в районе её полюсов падают в атмосферу;
3) Водород достаточно лёгок, его УФ выдувает из атмосферы, гелий - тяжелее и остаётся.

В итоге по прошествии длительного промежутка времени получим атмосферу с большим процентом гелия.
Аноним 27/11/17 Пнд 13:16:07  362346
>>362345
Как собираешься ТВЁРДО описывать физику червоточин, затраченную на них энергию и астро инженерию у звезды О класса, просто весь этот жесткий спектр и на электронику влияет не очень то.. Вот всё равно же найдутся те кто скажет не достаточно твёрда
Аноним 27/11/17 Пнд 13:21:58  362347
>>362346
Физику червоточин я мягко обойду, т.к. полагаю, что науке ещё многое неизвестно.

Про астроинженерию - на электронику жёсткий спектр влияет плохо, да, но это, всё же УФ, а не рентген. Даже металлическая фольга - надёжная защита. Ну и легированное стекло/легированный кремниевый монокристалл (окошки разнообразных датчиков).

В любом случае, спасибо за критику, критика только делает мой, кхм, набросок, твёрже :3
Аноним 27/11/17 Пнд 14:26:55  362356
>>362345
1000 а. е. это больше 5 световых дней. Думаю, хороший источник излучения на таком расстоянии от звезды можно заметить уже нашими средствами, если звезда не очень далеко от нас.
Аноним 27/11/17 Пнд 14:56:23  362361
Как часто СМЧД сливаются, если брать всю видимую вселенную? Раз в неделю? раз в 400 лет?
Аноним 27/11/17 Пнд 15:05:01  362363
>>362339
Твое перетекание не противоречит термодинамике?
Аноним 27/11/17 Пнд 15:19:08  362365
>>362363
Нет. Просто происходит выравнивание концентраций ядер гелия и водорода.
Аноним 27/11/17 Пнд 15:20:04  362366
>>362356
Может быть.. но там же туманность вокруг из выбросов от звезды, газ, нагретый излучением, светит в ИК и т.п..

Вон попробуй в туманности Гомункул разглядеть планету.
Аноним 27/11/17 Пнд 15:23:52  362367
image.png (138Кб, 532x900)
Смотри, анон
http://www.freepatent.ru/patents/2351512
есть типа системы для посадки капсулы на авторотации, но капсула там около 600. А что если приделать здоровенный винт на первую ступень по аналогичной схеме?
Аноним 27/11/17 Пнд 15:37:06  362368
>>362367
https://www.youtube.com/watch?v=MvMcsJTOYI0

Разобьется к хуям. Плюс, авторотация не по вертикали совершается, там вертикальная скорость гасится за счет горизонтальной, а тебе надо карандаш на попа посадить. Но еще раньше сгоришь при входе в атмосферу. А еще подозреваю, надо пилить полноценный механизм перекоса для управляемой посадки и огромные лопасти, все это по весу еще хуже складных крыльев.
Аноним 27/11/17 Пнд 16:36:17  362370
>>362314
>Теперь я учёный?
Сорян, ты опоздал на пару веков. Все уже попробовано до тебя. Остается только лизать космические камни.
Аноним 27/11/17 Пнд 16:38:04  362371
>>362316
>А если будет красная звуезда, небо будет зелёное?
Знаешь, на самом деле меня тоже волнует этот вопрос. Все эти кислотные небеса и проч. Но вроде как цвет определяется в первую очередь атмосферой. Видимый диапазон спектра у всех звезд одинаковый.
Аноним 27/11/17 Пнд 16:59:13  362377
>>362371
Диапазон-то одинаковый, а распределение интесивностей в нем -- нет.
Аноним 27/11/17 Пнд 17:39:42  362379
>>362377
Тогда объясни, при каких условиях небеса на планете будут как в КрузисеСпоре?
Аноним 27/11/17 Пнд 17:53:03  362382
>>362379
Ну хз. Зеленых звезд не бывает, потому что если пик светимости выпадает на зеленый, то синий и красный будут иметь практически такую же интенсивность, давая практически белый. Рэлеевское рассеяние в атмосфере всегда дает синий (более коротковолновый из видимого). Единственный вариант, который я могу придумать, это зеленый, но прозрачный, газ в атмосфере (в земной атмосфере все газы бесцветны). Вроде бы, тут подходит хлор.
Аноним 27/11/17 Пнд 18:05:44  362385
>>362382
Вот потому у меня на планете на орбите звезды O-класса небо сине-фиолетовое.
Аноним 27/11/17 Пнд 19:03:09  362396
926.970.jpg (231Кб, 800x800)
>>362316
>хочу увидеть небо всех цветов.
Легко.
Аноним 27/11/17 Пнд 19:16:57  362398
>>362382
А фиолетовый невидимый?
Аноним 27/11/17 Пнд 19:47:35  362399
>>362398
Цвет и прозрачность это разные вещи, мань.
Аноним 27/11/17 Пнд 19:54:52  362401
>>362389
что за 2 люка сверху на кабину у шаттла? Неужели оттуда в открытый космос выходили?
Аноним 27/11/17 Пнд 20:03:01  362403
А ведь майнить бетховен в космосе наверно охуенно профитно, ни пыли, ни воздуха, ни тепла, ваще кайф. Жаль что я не космонафт.
Аноним 27/11/17 Пнд 20:08:09  362404
>>362403
>ни тепла
шта?
Аноним 27/11/17 Пнд 20:24:07  362406
aHR0cDovL3d3dy5[...].jpg (93Кб, 660x437)
>>362401
Там на лётных Шаттлах два окна. Просто на Энтерпрайзе вместо них заглушки, потому что нахуя ему там окна.
>Неужели оттуда в открытый космос выходили?
Кто ж по-твоему всю кабину с полным экипажем будет разгерметизировать, чтобы в космос выйти? Это тебе не нанобанка на два-три огурца.
Аноним 27/11/17 Пнд 20:34:36  362414
>>362406
Я думал, там отдельное помещение может быть. А как они выходили? Через грузовой отсек?
Аноним 27/11/17 Пнд 20:38:58  362416
>>362406
И почему у шаттла такой здоровенный нос? У него мама еврейка? Там такие большие компьютеры спрятаны и прочая авионика?
Аноним 27/11/17 Пнд 20:47:51  362417
453398mainwhats[...].jpg (1278Кб, 3008x2000)
RetirementDisco[...].jpg (453Кб, 1100x733)
>>362414
Да, только между ГО и кокпитом ещё был очевидный шлюзовой отсек.
>>362416
Там передний блок маневровых двигателей.
Аноним 27/11/17 Пнд 20:56:29  362418
>>362339
Червоточины - для быдла. Собираем черную дыру такой массы, излучение Хокинга которой уже позволяет ее дозаряжать обычной материей. Выводим несколько таких на орбиту вокруг общего центра масс. Строим вокруг получившейся штуки сферу Дайсона. Летаем на такой штуке по всей галактике, подкармливая черные дыры встречающимися на пути звездами и планетами.
Аноним 27/11/17 Пнд 21:06:11  362420
>>362347
>что науке ещё многое неизвестно.
Это уже не твердая НФ, имхо. Твердая допускает, что инженерам что-то еще неизвестно, но не ученым.
Аноним 27/11/17 Пнд 21:20:33  362426
>>362399
Ну так фиолетовый более коротковолновый из видимого спектра, чем синий же.
Аноним 27/11/17 Пнд 21:26:12  362428
>>362426
Рассееваются все цвета, но чем короче длина волны, тем сильнее. Больше всего рассеивается действительно фиолетовый, но если "собрать" все, то получается сине-голубой. Ну типа так (числа условны):
фиолетовый: 100
синий: 80
голубой: 55
зеленый: 40
желтый: 28
оранжевый: 18
красный: 10
Если взять средневзвешенное, то получится примерно то, что мы видим на небе.
Аноним 27/11/17 Пнд 21:40:17  362429
>>362428
Кстати, на Марсе атмосфера красноватая, потому что рэлеевское рассеяние особой роли не играет (атмосфера чахлая), зато полно мелкой пыли бурого цвета. Теоретически, если пыль зеленая (наверняка есть подходящие минералы, их же дохуя), то и атмосфера может быть зеленоватой.
Аноним 27/11/17 Пнд 21:41:21  362430
>>362417
Ого, спасибо
Аноним 27/11/17 Пнд 22:43:13  362435
Вот представим, что гидратное ружьё выстрелило, и атмосфера земли начинает медленно превращатся в винерианскую. Допустим человечество не растерялось и решила дегазировать планету ака тероформировать. Есть ли такая возможность построить заводы по дегазации атмосферы?
Аноним 27/11/17 Пнд 22:49:05  362436
029 (1).jpg (53Кб, 800x520)
>>362382
Так речь не про цвет звезды. Ты чем читаешь? Я про цвет неба как составляющую всего - химсостава атмосферы, времени суток на планете в точке положения наблюдателя, тип и класс звезды, её удалённость от планеты и т.д. и т.п. И всем прекрасно известно что нет зелёных звёзд, что бывают красные, оранжевые, жёлтые, белые и голубые, но каких-нибудь синих, фиолетовых, розовых и зелёных. Но это я про самоцвет светила, а в зависимости от планты, выглядет она будет всегда по разному. Вот я и спрашиваю - в каких условиях будут возможны цветные небеса?
Аноним 27/11/17 Пнд 22:54:06  362437
>>362435
Да ну, просто помещаем в L1 экран от солнечного света, а потом собираем метан руками.
Аноним 27/11/17 Пнд 23:00:58  362438
>>362436
Цвет звезды влияет на цвет неба, улавливаешь? Но зеленых звезд не завезли, поэтому надо по-другому извращаться. Я придумал только зеленый газ в составе атмосферы (возможно, хлор), ну и зеленую пыль.
Аноним 28/11/17 Втр 01:08:49  362449
На Луну щас какие ни будь говна падают? Кометы там или астероиды, ток большие что бы. Если да, то есть ли видосы как это все дело происходит. Чтоб прям охуенно было, как в кино.
Аноним 28/11/17 Втр 01:38:08  362450
rendition.mp4 (1226Кб, 480x270, 00:00:19)
>>362449
Солнечная система очень, очень большая, Луна очень, очень маленькая, астероиды ещё меньше. Можешь сам посчитать вероятность и сколько потребуется времени. Хотя недавно упала козявка.
Аноним 28/11/17 Втр 02:11:28  362453
>>362437
Я к тому что, есть ли у нас столько химикатов чтобы проводить реакции в масштабах планеты? Да и не выбросится при сегодняшних технологиях хуйни всякой парниковой больше, чем весь метан?
Аноним 28/11/17 Втр 03:17:02  362455
image.png (31Кб, 907x291)
>>362297
Да, в 10 я слепошарый

Куб воздуха 0.318, гелия - 0.0439. Положительная плавучесть 0.2741 грамм с куба, тогда на шар с полезной массой в общей сложности в 3 тонны надо 1 095 000 кубов = шар в 65 метров радиусом.

Но тогда получается забраться на 70-80 километров теоретически реально хотя бы на небольшом метео аэростате с градусником и чашкой для сбора осадков.
Аноним 28/11/17 Втр 03:19:15  362456
А японцы только 53 километра взяли.
Аноним 28/11/17 Втр 06:55:13  362462
>>362361
Реже. СМЧД есть обычно только в центрах галактик, а те сливаются ОСНЕ редко.
Аноним 28/11/17 Втр 06:57:48  362463
>>362403
Со сбросом тепла там жопень если что. Чай в термосе по этой же причине не остывает.
Аноним 28/11/17 Втр 07:00:12  362464
>>362429
Окислы меди или плазма бора.
Аноним 28/11/17 Втр 07:38:33  362466
>>362403
Тепло куда сбрасывать будешь?
Аноним 28/11/17 Втр 09:05:52  362473
Если я выйду в открытый космос со снарягой аквалангиста, то что со мной случится? Допустим на теневой стороне земляшки, чтобы не рассматривать воздействие солнца.
Аноним 28/11/17 Втр 10:08:25  362481
>>362473
Раздует. Воздух из очков-маски выйдет, закипят слизистые. Воздух начнет выходить через ноздри.
Аноним 28/11/17 Втр 10:12:59  362482
>>362481
>раздует
Афаик куски мяса довольно прочные, и это ничем страшным им не грозит.
>воздух из под маски выйдет
А если резинку поплотнее?
Аноним 28/11/17 Втр 10:13:17  362483
>>362449
>На Луну щас какие ни будь говна падают?
Регулярно
>Кометы там или астероиды, ток большие что бы.
Большие давно уже упали. Сейчас вероятность падения большого булыжника крайне мала. Километровые раз в 200 тыс лет, десятикилометровый раз в 100 млн лет.


Аноним 28/11/17 Втр 10:33:21  362487
>>362418
Чем ты их удержишь в нужной позиции, уважаемый? [Глубоко гипотетическим] генератором гравитации?
Аноним 28/11/17 Втр 10:34:28  362488
>>362420
Эх, значит у меня несколько неправильное понимание твёрдой НФ.
Аноним 28/11/17 Втр 11:00:49  362491
>>362450
>>362483
Спасибо.

> Сейчас вероятность падения большого булыжника крайне мала
Эх жаль, хотелось посмотреть на какую ни будь йобу по типу той что огромный кратер оставила с лучами. Кстати это давно было и че там упало, ну и каковы последствия.
Аноним 28/11/17 Втр 12:30:28  362549
>>362487
Даже если цивилизация не найдет аналитического решения, при наличии достаточной вычислительной мощи и времени, вполне можно найти какое-нибудь численное решение задачи n тел, при котором эти тела не сталкиваются и не вылетают за очерченные границы в течение ближайших T лет. У цивилизации, живущей на сфере Дайсона, вычислительные возможности даже без компьютеров внушительные. Если это необходимо, орбитальные параметры ЧД поправляются соответственно.
Аноним 28/11/17 Втр 12:36:45  362562
трителаорбиты.webm (71Кб, 400x400, 00:00:20)
>>362549
Правда, ощущаю проблему с самой сферой Дайсона. Она, возможно, будет немножко ходить ходуном
Аноним 28/11/17 Втр 13:45:14  362654
>>362549
>орбитальные параметры ЧД поправляются соответственно.

Вот я и спрашиваю, как ты их поправлять будешь? Двигатель к ЧД не приделать.. Будешь подгонять к заданной точке коричневый карлик, чтобы он своей гравитацией орбиты подправил?
Аноним 28/11/17 Втр 15:49:48  362774
111111111111111[...].png (54Кб, 1440x900)
Два вопроса про гравитации.
1) Почему же они всё так не падают?
Гравитация изображается как колодец. И тем она больше, чем массивнее, а не объёмнее теле. И что вращение планет - типа скатывание шариков в воронку. Но почему же тогда планеты стабильны? Почему они скатываются ко дну гравитационного колодца? Почему Юпитер не скатывается в Солнце? С одной стороны визжат про пердел Роршаха, и что вот-вот, через пару миллионо-триллионов лет, и Луна на Землю упадёт, при этом другие (а иногда и эти же) визжат о грядущей катастрофе из-за того, что Луна вот-вот улетит на совсем, вода обезумит, динамо сломается и т.д. Какое-то это всё наебалово.
2) Задача трёх тел
Нет никакой Глории по ту сторону Солнца или власти в сговоре с учёными и, как говорится, скрывают..., там есть только точка Лагранжа. Но почему всё же не может быть второго тела на той же орбите? Спутники (не в смысле планеты) спокойно ютятся по десяткам и сотням. И не говорите, что они маленькие, а у планет появляются Лагранджи. Почему они, собственно, так появляются? Что им мешает распологаться плотнее типа как пикрелейтед?
Аноним 28/11/17 Втр 15:59:54  362785
>>362774
>изображают как воронку
Просто хоть какая-то визуализация пространства, близкая для понимания рядовому васяну. Ирл оно не так.
>чому юпитер не падает на солнце
Чтобы упасть, ему нужно затормозить на своей орбите. А орбитальную скорость у него изменить ему изменить нихуя непросто.
Аноним 28/11/17 Втр 16:05:37  362792
>>362774
Они падают, но постоянно промахиваются
Аноним 28/11/17 Втр 16:26:31  362800
>>362785
>визуализация пространства
Не пространства, а поля. Гравитация это не искажение пространства, а изменение траектории под действием физического поля, пространство же остается нормальным. Если я не прав, то давай пруфы что исказилось именно пространство, а не траектория.
Аноним 28/11/17 Втр 16:34:25  362803
>>362800
Фотоны же, они не имеют массы в привычном понимании (хотя у них есть "масса импульса" или что то подобное, иначе бы никакого давления света не существовало), но тем не менее возле источников сильной гравитации появляются гравитационные линзы, которые загибают их траекторию полёта.
Аноним 28/11/17 Втр 16:48:34  362807
>>362803
В обычных линзах вот фотоны тоже меняют траекторию, но ни у кого нет подозрения там искажается пространство. Как обнаружить именно искажение пространства и отличить его от всего другого?
Аноним 28/11/17 Втр 17:13:45  362822
>>362473
Клапана потравит, задохнёшься.
>>362482
>А если резинку поплотнее?
А так - раздует, будет больно как от засоса, только по всему телу. Пердёж пойдёт, очко тоже потравит. Любопытно, высосет ли желудочный сок через жопу и станет ли воздух травить через ЖКТ?
Аноним 28/11/17 Втр 18:18:06  362857
>>362807
Ну насколько мне известно, то подсчётами. Вот у нас есть обычное притяжение света к большому объекту, и в добавок этот объект искажает метрику. В итоге свет загибается сильнее чем если бы в евклидовом пространстве притягивался бы свет.
Аноним 28/11/17 Втр 18:49:23  362859
>>362487
Саморезами.
Аноним 28/11/17 Втр 19:03:00  362861
>>362774
1)
"Типа скатывания шариков в воронку" именно что типа, это не полная аналогия, а лишь примерная. Альзо, если шарики катятся без трения, то в воронку они никогда не упадут, а так и будут мотать круги вокруг нее. Приближение/удаление возможно за счет приливных сил. Грубо говоря, Луна замедляет вращение Земли, а высвободившаяся энергия на децл ускоряет Луну, делая ее орбиту все выше и выше. При другом соотношении параметров может быть и приближение.
2) Это возможно теоретически, но такая орбита нестабильна: малейшее возмущение вытолкнет твою антиземлю из противоположной точки, а дальше притяжение уже не даст ей туда вернуться. Как дальняя аналогия: яйцо можно сбалансировать так, чтобы оно "стояло вертикально", но малейший толчок его выведет из этого положения, а само оно туда не вернется.
Аноним 28/11/17 Втр 19:10:10  362865
"Магической" является только скорость света в вакууме c, так? То есть, если в каком-нибудь веществе свет движется со скоростью 0,5c, ничто не мешает какому-нибудь нейтрино лететь в нем на 0,9c?
Аноним 28/11/17 Втр 19:49:57  362880
поляризатор.jpg (942Кб, 1261x842)
Угораю по микрофотографии. На микроскопе установлены 2 поляризационных фильтра. Чому противоположные четверти заднего фона окрашены одинаково? Значит ли это, что вектора амплитуды световой волны совпадают с моими зеленой и синей стрелочкой, и как раз перпендикулярны друг другу? Но мне кажется это сомнительным, ибо после фильтров свет же должен быть линейно поляризован, не?
Аноним 28/11/17 Втр 19:50:16  362881
>>362865
Ты только что излучение Черенкова.
Аноним 28/11/17 Втр 20:00:22  362885
>>362881
Почитал, спасибо.
Аноним 28/11/17 Втр 20:52:29  362911
>>362880
Ах да, забыл сказать, что там ещё стоит гамма-компенсатор.
Аноним 28/11/17 Втр 21:22:19  362933
>>362654
Ровно так же, как предлагается перемещать звезды и всю систему в целом.
https://ru.wikipedia.org/wiki/%D0%97%D0%B2%D1%91%D0%B7%D0%B4%D0%BD%D0%B0%D1%8F_%D0%BC%D0%B0%D1%88%D0%B8%D0%BD%D0%B0
Чем меньше масса ЧД, тем быстрее она испаряется. Небольшие ЧД светят так, что в них из-за давления электромагнитного излучения очень сложно закинуть материю. Мы же имеем ЧД такой пограничной массы, при которой сила тяжести вблизи горизонта событий немного превосходит давление электромагнитного излучения, или даже немного меньше, но при этом ее еще относительно легко перезаряжать. Люто светить при этом она все еще будет, а значит ее можно перемещать с помощью звездной машины.
Аноним 28/11/17 Втр 21:24:05  362935
>>362933
Еще вдогонку статья про использование ЧД в качестве двигателя:
https://ru.wikipedia.org/wiki/%D0%A1%D0%B8%D0%BD%D0%B3%D1%83%D0%BB%D1%8F%D1%80%D0%BD%D1%8B%D0%B9_%D1%80%D0%B5%D0%B0%D0%BA%D1%82%D0%BE%D1%80
Аноним 28/11/17 Втр 23:57:48  362975
Есть ли в солнечной системе спутники, не находящиеся в приливном захвате со своей планетой?
Аноним 29/11/17 Срд 00:07:48  362977
>>362975
Согласно вот этому списку:
https://en.wikipedia.org/wiki/List_of_gravitationally_rounded_objects_of_the_Solar_System, крупные все в приливном захвате. Но наверняка есть какие-нибудь мелкие и очень дальние спутники Юпитера или Сатурна подходящие.
Аноним 29/11/17 Срд 00:10:25  362979
>>362975
Нерегулярные спутники Юпитера и Сатурна, например.
Аноним 29/11/17 Срд 00:18:32  362980
>>362975
Например, Нереида (у Нептуна).
Аноним 29/11/17 Срд 04:17:26  363013
Что будет если к горизонту событий черной дыры поднести очень длинный шест, длинной скажем в миллион километров? Один конец шеста начнет падать в черную дыру, а что будет с противоположным концом, который не находится под воздействием черной дыры? С ним будет все тоже самое, он просто начнет исчезать? Если да то почему так?
Аноним 29/11/17 Срд 06:53:13  363015
>>363013
Такой шест легко ломается. Первый кончик просто отожрёт, внутри ГС шест не сохранит целостность ведь. Если не мешать, остальная часть шеста тоже потихоньку притянется, у самого конца раздираясь приливными силами (или чем оно там раздирает, гравитацией короче). Ну, если гипотетически допустить, что шест неломаемый или может бесконечно растягиваться, то да, второй конец притянется достаточно быстро и чд втянет весь шест. За ощутимое время, естественно, ибо растяжение и сила втягивания шеста в чд не может распространяться по шесту быстрее скорости света. Вообще, зависит от самого шеста. То есть, как я и сказал, если он прямо-таки почти бесконечно прочный, то втянуться должен очень быстро. Ибо разрушаться он должен начать лишь уже после преодоления горизонта событий, и вся сила, действующая на него около ГС, будет распространяться на весь остальной шест со скоростью звука в шесте. А если чуть рыхлее, то куски от шеста будут отдираться ещё на подлёте к ГС и время, за которое шест зохавает, будет уже больше. Чем рыхлее, тем больше. Если брать реальные условия, то есть вполне ограниченный по прочности шест, то он и притянется примерно за такое же время, за какое объект, находящийся в миллионе километров от чд, притянется к чд. Представь себе, например, звезду массой сорок Солнц. За какое время такая звезда притянет объект в миллионе км. от себя, за такое же время притянет и чд массой 40 солнечных. И второй конец шеста тоже ибо шест будет крошиться oche сильно. Где там мои расчёты были... Во, цитата, "на его нижнюю часть будет действовать ускорение 400000000000м/с2, а на верхнюю — в 1.00003(3) меньшая, то есть разница будет 13000000м/с2, ну или где-то 65 километров на секунду в квадрате на каждый сантиметр тела, вот кусочек плоти на сантиметр дальше, и уже разница в 65 километров в секунду через секунду". То есть есть вот у нас две пылинки у края ЧД, летят с одинаковой скоростью. Через секунду та, что на сантиметр ближе к ЧД, уже должна обрести скорость на 65км/с большую, чем та, что на сантиметр дальше. Представляешь себе силу разрыва и какой прочности нужен шест?
Аноним 29/11/17 Срд 07:17:34  363018
>>363015
>Представляешь себе силу разрыва и какой прочности нужен шест?

прочный как жопа битарда
Аноним 29/11/17 Срд 07:21:53  363019
>>363018
Между атомами кристаллической решетки объекта близ (не сверхмассивной) ЧД электромагнитные силы будут слабее гравитации, объект просто порвет на составляющие атомы.
А если пересечь горизонт, так и вовсе из-за скорости света не будет возможности атомам сообщаться друг с другом, а то и кваркам, даже слабые/сильные взаимодействия не смогут передаваться, объект распидорасит даже при пересечении горизонта СМЧД.
Аноним 29/11/17 Срд 07:44:19  363020
>>363019
Так, стоп, а как вообще притягиваются чёрные дыры к другим объектам? Говорят, из-за замедления времени частицы не долетают до горизонта событий с нашей точки зрения. Соответственно, гравитоны, которые должны передавать взаимодействие, тоже не будут долетать, а будут тормозиться у границы чд даже не имея возможности влететь в какие-нибудь другие частицы, не то что в саму чд. И как гравитоны передают гравитацию от чд к другим объектам если около неё время замедлено и они не могут покинуть её? Видимо, я что-то не так понимаю в этих ваших замедлениях.
мимо->>363015
Аноним 29/11/17 Срд 08:01:55  363022
>>363019
Не распидарасит на горизонте сверхмассивной, не пиши ерунды, приливные силы слишком слабы, а ещё не забывай что скорость света даже если летишь со скоростью близкой к с, все равно для тебя будет с.
Аноним 29/11/17 Срд 08:29:27  363026
>>363022
>Не распидарасит на горизонте сверхмассивной
На горизонте ЛЮБОЙ распидорасит. ГС это область где для пересечения ее наружу надо превысить скорость света. У тебя глюоны и электроны той части что пересекла ГС не сможет сообщаться с той частью, что не пересекла. Что удержит часть пересекшую ГС с той, что не пересекла?
А вот приливными силами близ СМЧД не распидорасит, это да, если ты это имел в виду.

>>363020
>гравитоны
Их существование не доказано, не спекулируй терминами из научной фантастики.
Черные дыры искажают метрику пространства-времени.
Аноним 29/11/17 Срд 08:51:31  363027
>>363026
Не распидарасит ибо в системе отсчёта тебя, падающего в дыру ты скорость света не достигнешь никак, ты и горизонт не заметишь
Если летишь со скоростью 0,99с и вытянув назад руку с фонариком посветишь себе в глаз то разницы во времени долета света до глаза не увидишь при таком же опыте на скорости в 0.
Аноним 29/11/17 Срд 08:56:02  363029
>>363027
>в системе отсчёта тебя, падающего в дыру ты скорость света не достигнешь никак
А ты и не должен, у тебя время замедляется, но падаешь ты с той же "скоростью" что и падал. Это для стороннего наблюдателя ты будешь долго падать, а для себя внезапно заметишь красоты https://www.youtube.com/watch?v=JcHneuh6DKo
а когда стнешь пересекать ГС, начнешь терять связь с частицами, пересекшими ГС.
Аноним 29/11/17 Срд 09:17:25  363030
>>363029
https://www.litmir.me/br/?b=265797&p=117 такую книжку читал?
Аноним 29/11/17 Срд 09:26:13  363031
berneydidnotread.gif (4505Кб, 413x273)
>>363030
Можно tl;dr?
Аноним 29/11/17 Срд 09:37:01  363032
Черные дыры и складки времени. Дерзкое наследие Эйнштейна
Автор:Торн Кип.
https://www.litmir.me/bd/?b=265797
Стр 117 и далее
Аноним 29/11/17 Срд 09:44:31  363033
>>363031
Впрочем, вопрос что будет если сунуть руку под горизонт, то почему её не оторвет все равно ломает моск.
Аноним 29/11/17 Срд 10:12:29  363034
>>362933
Понял тебя.

Не знаю, правда, насколько тверда эта идея - надо считать массу такой "пограничной" дыры, не будет-ли она слишком маленькой для звёздной машины.
Аноним 29/11/17 Срд 10:30:32  363036
>>362880
>>362911
Знаешь, не обращая внимания на то, что он называется телескопотред, советую тебе все таки с микроскопом пиздовать туда. Это не достаточно тупой вопрос. Он может и тупой в контексте, но свормулирован громоздко, что мне что-то даже впадлу вникать в него. Могу лишь сказать, что как он должен быть поляризован нужно спрашивать у твоих фильтров, мкей?
Аноним 29/11/17 Срд 10:44:17  363037
>>363036
Пора смириться с тем, что спейсач это место для вопросов по физике без клоунады саентача.
Аноним 29/11/17 Срд 10:54:49  363040
>>362297
Эта "оболочка из плазмы" нихуя не будет держать внутри. Технология, насколько я понимаю, давно используется в противоракетной обороне - на пути полёта сверхзвукового объекта создаётся плазменное поле, в котором природа сопротивления другая, не такая, как в воздухе. Для объекта, вроде бы обтекаемого в воздухе (а на самом деле уже не важно), это ощущается как кирпич. Чем выше скорость - тем хуже для объекта. То, что боинг рисует, выглядит странно - может быть, поля хватит, чтобы затормозить пули, но таскать с собой такую установку это пиздец, не ядерный же реактор на каждый танк пихать, там мегаватты нужны.
Аноним 29/11/17 Срд 11:20:08  363042
>>363036
Там стоят обычные линейные поляризаторы. Свет выходит из лампочки, проходит через поляризатор, гамма-корректор, конденсор, попадает на предметное стекло и в объектив, а после него стоит анализатор и окуляр. Анализатор статичен, и вращается только поляризатор. Гамма-корректор не влияет на характер затемнения четвертями как на пикче, только что проверил. Но в телескопотреде все равно отписался.
Аноним 29/11/17 Срд 11:26:40  363043
>>363026
>ГС это область где для пересечения ее наружу надо превысить скорость света
Вот мне уже давно покоя один вопрос не даёт. ГС это где вторая космическая равна скорости света, то есть скорость обращения вокруг ЧД будет равна цэ. Но ведь и чтобы вылететь с планеты не обязательно развивать 7.9км/с, достаточно будет лететь и на скорости 1 метр в секунду постоянно газуя пока в конце концов не улетишь. Если из-под горизонта очень сильно газовать то разве не получится и оттуда улететь?
>Их существование не доказано
Как так? Или там пока только гравитационные волны доказали и я перепутал их с гравитонами?
Аноним 29/11/17 Срд 11:35:26  363044
Как бы выглядела растительность при разных спектральных классах звезд?
Аноним 29/11/17 Срд 11:41:34  363045
>>363044
Вряд ли она бы жила в привычном виде под разными классами.
Аноним 29/11/17 Срд 11:53:52  363046
1354583547320.jpg (207Кб, 1280x700)
>>363026
>части что пересекла ГС не сможет сообщаться с той частью, что не пересекла.
Нет модели, явно описывающей физическое состояние тела под горизонтом событий. Есть гипотезы, ценность которых для науки на данный момент сомнительна, но пусть будут. Конкретных имен и ссылок нет. Мат аппарат для такой гипотезы пока не представляется разрабатываемым. Но суть в том, что с учетом даже не гравитационного, а просто релятивистского замедления времени(ибо предполагается, что объект ныряет под гс на околосветовых скоростях), ничто не мешает сохранить составным частицам связь между собой и практически не изменить телу формы. Поэтому за короткий период перехода через гс вовсе необязательно, что форма тела будет изменена или полностью разрушена. Теория поля в стандартной модели вполне подразумевает колебания интенсивности полей взаимодействий из-за квантовых эффектов даже с учетом констант в формулах. Это означает, что при закритических параметрах метрики, которые не вообразимы в бытовом представлении, интенсивность внутриатомных, молекулярных и т. д. связей не постоянна, в дальнем приближении представляется функцией гармонических колебаний. Поэтому сам процесс перехода через ГС является лишь частным случаем такого непостоянства, без обязательных "шредерных" последствий для материи. Все это справедливо, офк, для сверхмассивных дыр. И по примитивным моделям, вытекающем из вышесказанного, на пути к сингулярности все равно распидорасит, потому что даже у смчд градиент нарастает к центру, и куда сильнее, просто функция этого приращения набирает обороты уже на расстояниях, меньших, нежели радиус ГС.

На базе такого представления строятся совсем уж омские истории вроде планет под ГС, и вполне уважаемыми мужами строятся. А смешно то, что даже такую дичь опровергнуть современный аппарат не может. Ну и для экзотики можно вспомнить про червоточины, про "окна" в другую вселенную, про белые дыры на другом конце чего-нибудь и прочее.

Да, гс - граница, между которой нет никакого сообщения. Но не значит, что это шрёдер. Отрезанный палец можно и пришить. А если отрезать и пришить за несколько триллионных долей секунды, то жертва может и не заметить. Это все, конечно, та еще лирика, и к реальности мало отношения имеет. Но гипотеза есть, опровергающих ее выводов нет. Так что ты в заблуждение людей не вводи с глюонами-электронами.
>Их существование не доказано, не спекулируй терминами из научной фантастики.
В данном контексте куда важней, что их существование не опровергнуто. Блять, есть даже гипотезы неинформационности имеющих массу гравитонов, безмассовых гравитонов, гравитонов, как квазичастиц. Стандартная модель слишком сладка, чтобы насовсем бесповоротно от нее отказаться.
>Черные дыры искажают метрику пространства-времени
Очередной кататель шаров по натянутым тряпкам. А что ты знаешь про нее, про метрику то? Хоть одну компромиссную мат-модель осознаешь? Учи квант мех, или пасть не разевай, визионер.
Аноним 29/11/17 Срд 12:16:25  363047
>>363043
Чисто гипотетически если материальная точка, находящаяся под гс, движется с конкретной скоростью по вектору, противоположному направлению на сингулярность, то она покинет горизонт событий. Но это на бумаге онли. В реальности скорость падения материи в дыру околорелятивистская. Чтобы сменить вектор, потребуется нереально много энергии. Гипотетический движок, который может развить такой внезапный импульс в компактном представлении, сам уже, одним этим импульсом создаст закритические параметры метрики и провалится в себя, сколлапсировав в чд по принципу эквивалентности энергии-массы прежде, чем куда либо уедет.
Аноним 29/11/17 Срд 12:25:43  363049
>>363047
Вот как.
Аноним 29/11/17 Срд 12:38:08  363051
>>363047
Сингулярность доя пидоров.
Аноним 29/11/17 Срд 12:57:03  363056
>>363043
>на скорости 1 метр в секунду постоянно газуя
Ты не сможешь лететь на скорости даже метр в секунду против гравитации черной дыры под горизонтом событий, для этого нужна бесконечная энергия. Твой пример аналогичен примерно такому: "мне ж не надо лететь на скорости 10c, достатчно c плюс 1 метр в секунду, ну а метр в секунду это немного, просто надо чуток газануть".
Аноним 29/11/17 Срд 12:57:46  363058
>>363051
Я сам за планковскую звезду топлю, но хуле от того?
Аноним 29/11/17 Срд 13:35:26  363063
>>363058
Не использовать понятие сингулярности.
Аноним 29/11/17 Срд 15:58:06  363096
>>363047
>закритические параметры метрики
Где про это почитать?
Аноним 29/11/17 Срд 18:51:28  363126
Насколько обосновано предположение о самовозгорание атмосферы при ЯО? Вроде где-то слышал что 300 мт и кирда. Но мне почему-то кажется, что это пиздёшь.
Аноним 29/11/17 Срд 18:59:53  363127
>>363126
Настолько же обосновано, что ритег может взорвать газовый гигант.
Аноним 29/11/17 Срд 22:01:08  363159
>>363044
https://elementy.ru/nauchno-populyarnaya_biblioteka/430636/Tsvet_rasteniy_na_drugikh_planetakh
Аноним 30/11/17 Чтв 00:41:34  363178
>>363046
>пик
Идеальная технология блет!
Аноним 30/11/17 Чтв 13:16:23  363224
>>363046
>На базе такого представления строятся совсем уж омские истории вроде планет под ГС
Так там и звезды тогда могут светить, лол.
Аноним 30/11/17 Чтв 13:35:33  363228
image.png (31Кб, 406x268)
>>363047
>Чисто гипотетически если материальная точка, находящаяся под гс, движется с конкретной скоростью по вектору, противоположному направлению на сингулярность, то она покинет горизонт событий.
Да ну нахуй. Ни одна из временных линий не покидает горизонт. У тебя на диаграмме движение со скоростью света это 45 градусов. Оказавшись за ГС ты можешь его покинуть только со сверхсветовой скоростью.
Аноним 30/11/17 Чтв 14:03:07  363231
>>363126
>Насколько обосновано предположение о самовозгорание атмосферы при ЯО?
Тогда нихуя не знали особо. Речь шла не о горении (окислении топлива), т.к. если бы атмосфера могла гореть, она бы уже это сделала. Речь шла о цепной реакции, когда от сверхмощной вспышки начали бы атомы атмосферы фьюзиться. Только это не так, т.к. если бы могла быть цепная реакция, она бы пошла от какой-нибудь ОМГ частицы.
>пиздёшь
Впервые вижу чтобы в слове "пиздёж" допускали две ошибки. Песдетс, таваресчи.
Аноним 30/11/17 Чтв 14:18:36  363234
>>363231
>Песдетс, таваресчи.
Это всё потому, что человечество сейчас наблюдает пандемию РЕЦЕССИИ И СТАГНАЦИИ у большей части населения. Сам я ведь тоже болен... сука блят!
Аноним 30/11/17 Чтв 14:26:28  363235
>>363234
Нет вот реально, вот как блет прекратить жрать говно, находить эстетику в пустоте простых структурах материи, которым не нужно испытывать пиздец жизни и продолжить уже эволюционировать усложняясь дальше, почему желание принять свою дефективность и дожить срок не тратя лишней энергии на всякие превозмогания развития, НАСТОЛЬКО БЕЗУМНО СИЛЬНОЕ этож пиздец какой то... Как вообще хоть кто то с этим справился в принципе, начиная от тех кто покинул тёпленькие места в Африке и заканчивая теми кто сейчас пытается разработать детальный план колонизации Марса?
Аноним 30/11/17 Чтв 14:41:14  363236
>>363235
.... Столько знаний, столько разработок, столько способов коммуникации и организации данных, колоссальный объём информации, идей, мнений, концепций, инженерных конструктов... Ваши сингулярности, я же почти ничего не понимаю в том как это всё создали и как это повторить с нуля, я же тупил бы тысячелетиями если реально было бы вообще столько прожить как полный даун в попытках осмыслить всё то чем владеет человеческая цивилизация и понять как этим всем оперировать, вот уж я действительно лысая обезъяна, я чувствую себя настолько примитивным и тупым... Похоже я просто очередная дефектная ветвь эволюции, подлежащая отсеиванию по естественному отбору не способная адаптироваться к изменяющимся условиям и никакие психологические манякомпенсирования уже никогда не отменят моего понимания этого. Мне остаётся только ждать конца в крайне низко эффективных попытках стать не отсеивающимся эффективность которых будет в сотни раз меньше, чем у любого нормального индивидуума, а так же пытаться приносить пользу работая наработу, эффективность которой тоже будет неимоверно посредственной... Вот такая вот ахуительная история под названием существование и это его истина для меня, пиздетс.
Аноним 30/11/17 Чтв 16:08:11  363245
>>363047
Ну насколько я знаю из видео из спейсенжина, когда ты переходишь гс то со всех сторон на тебя будет смотреть одна большая сингулярность. К тому же как ты пересечёшь второй гс под поверхностью?
Аноним 30/11/17 Чтв 17:19:40  363249
>>363231
>Впервые вижу
Не пизди, ты просто не обращал внимания.
Аноним 30/11/17 Чтв 19:10:53  363260
Что за хует про графен с энергией из ничего
не является ли это наебкой второго закона термодинамики?
на энергию из нихуя похуй а вот прямая конвертация температуры в микроэнергию это же охуенный мегапрорыв в космонавтике!
Аноним 30/11/17 Чтв 19:18:55  363262
>>363260
>хует про графен с энергией из ничего
Может ты поподробнее напишешь про это, а то я нихуя не понял.
Аноним 30/11/17 Чтв 19:23:54  363263
>>363262
https://www.popmech.ru/science/398982-grafen-okazalsya-istochnikom-beskonechnoy-energii-revolyuciya-v-energetike/
Аноним 30/11/17 Чтв 19:24:20  363264
>>363262
Он просто придумал какую-то хуету, а теперь сам агрится.
Аноним 30/11/17 Чтв 19:25:26  363265
>>363263
>числа энергии
Мда, хех.
Аноним 30/11/17 Чтв 19:29:50  363266
>>363264
нихуя не знаешь но всюду лезешь?
Аноним 30/11/17 Чтв 19:43:00  363269
>>363266
Я знаю, что по-русски так не говорят, например.
Аноним 30/11/17 Чтв 20:02:23  363271
15112893803260.jpg (102Кб, 802x747)
>>363245
>сингулярность
Аноним 30/11/17 Чтв 20:36:40  363274
>>361165 (OP)
Есть ли реальные фотографии черных дыр ? Или обоснованные доказательства(Мне лен искать в интернетах)
Аноним 30/11/17 Чтв 20:43:35  363276
>>363274
Есть
Аноним 30/11/17 Чтв 20:47:46  363277
>>363276
Он не про твое очко спрашивал.
Аноним 30/11/17 Чтв 21:03:05  363280
Почему без Шаттла нельзя слетать к какому-нибудь тому же Хабблу и починить его? Кто мешает подлететь на Союзе, вылезти из него и поковыряться?
Аноним 30/11/17 Чтв 21:11:20  363281
>>363280
Союз это банка 2 на 3 метра без возможности вкд. Там ни вздохнуть, ни пернуть. Шател огромная еба на семь обезьян, с вкд и местом даже для ебли.
Аноним 30/11/17 Чтв 21:19:38  363283
https://hightech.fm/2017/11/24/graphene_energy
хуле тут не понятно, тупарики?
тепло научили перетекать в энергию напрямую. Пусть энергии будет мало, главное чтобы тепло исчезало внутри системы.
конечно они могут пиздеть, но там пиздунишек не любят и анус свой они уже поставили.
и тогда можно будет юзать ЯЭДУ
Аноним 30/11/17 Чтв 21:20:51  363284
>>363281
> без возможности вкд
Врать зачем?
>На 35-м витке космонавты Хрунов и Елисеев вышли в открытый космос из корабля «Союз-5» и перешли в корабль «Союз-4».
https://ru.wikipedia.org/wiki/Союз-5
Бытовой отсек может служить шлюзом.
Да, банка мелкая. Но положите запчасти в скамейку бытового отсека и посадите двух макак вместо трех, будет удобно.
Почему кроме "нетзадач" так не стали делать?
Аноним 30/11/17 Чтв 21:26:58  363285
>>363284
Разговор идёт про сейчас, а не про было когда-то.
А запчасти для хаббела просто засунуть не получиться. Да и двух обезьян точно не хватит.
Аноним 30/11/17 Чтв 21:31:33  363286
>>363285
Необязательно хаббл, само собой.
Ну и можно джва союза отправить, состыковать, как 4 и 5, будет больше макак и места.
Да и запчасть если крупная какая можно отдельно отправить.
Я к тому, что фундаментально же возможно обойтись без шатала/бурана для подобных махинаций, чому нет?
Аноним 30/11/17 Чтв 21:36:47  363287
>>363286
Слишком много гемороя.
Аноним 30/11/17 Чтв 21:39:08  363289
>>363287
А если запустят на НОО новый телескоп Джон Вебстер и у него не раскроется зеркало, миллиард баксов затопят нахуй или все-таки попробуют слетать и вылезши из банки починить монтировочкой?
30/11/17 Чтв 21:50:08  363291
>>363289
В смысле JWST? Зеркало у него должно раскрыться только в L2 Солнце-Земля - хуй ты уже туда долетишь.
Аноним 30/11/17 Чтв 21:51:56  363292
>>363291
Нет, не Джеймс Вебб. Какой-нибудь другой дорогостоящий спутник. Потому и переназвал и написал про НОО.
Аноним 30/11/17 Чтв 22:00:14  363293
>>363292
Если что могут пульнуть Орион. Вообще НАСА планирует новый видимый телескопчик в 20х.
Аноним 30/11/17 Чтв 22:03:42  363295
>>363293
Я всячески не слышу ответа на изначальный вопрос.
Тогда перефразирую.
Альтернативная реальность, наше время.
Шаттлы всё, союзы ещё, орион откладывают и откладывают и полетит он года через три в лучшем случае.
Роскосый совместно с Насой запускают йоба-телескоп за гигабакс круче Хаббла на низкую ололоземную.
И тут выяснилось, что опять обосрались с зеркалом или монтировкой вторичного зеркала - все решается небольшой юстировкой вторичного зеркала.

Смогли бы полететь на союзе чинить?
Аноним 30/11/17 Чтв 22:04:55  363296
>>363291

>>363292
Ну или ATLAST, если его все таки запилят
Аноним 30/11/17 Чтв 22:06:58  363298
>>363296
>ATLAST
>НАКАНЕЦТА
Его будут откладывать и откладывать, чтобы при запуске он оправдал своё название, лол.
Аноним 30/11/17 Чтв 22:08:20  363299
>>363292
У них есть 2 телескопа типа Хаббл от NRO. Из 1 сейчас WFIRST пилят.
Аноним 30/11/17 Чтв 22:10:24  363301
>>363295
>Смогли бы полететь на союзе чинить?
Нет. Теоретически Dream Chaser подготовят для этого.
30/11/17 Чтв 22:19:44  363302
>>363295>>363301
Смогли бы. Вопрос только в том, как сблизиться с телескопом.
Аноним 30/11/17 Чтв 22:20:21  363303
>>363302
Так же как с МКС.
30/11/17 Чтв 22:43:31  363308
>>363303
У МКС есть Курс. Ставить такую штуку на космический телескоп заранее как-то уж слишком предусмотрительно. Систему сближения Союза придётся модифицировать.
Аноним 30/11/17 Чтв 22:46:09  363309
>>363308
Зачем? Пусть ЦУП считает траектории.
В кербаче было довольно просто это делать.
Аноним 01/12/17 Птн 00:21:31  363312
>>363284
На ремонт хаблла ушло 5 полетов, в каждом примерно по 5 парных выходов по 7 часов. Это больше 300 часов работы в открытом космосе.

Я бы дорого заплатил за цирковой номер с Союзами.
Аноним 01/12/17 Птн 00:23:15  363313
>>363312
на ремонт и обслуживание*
СЛоЖНА Аноним 01/12/17 Птн 00:27:54  363315
Если черная дыра имеет бесконечную массу и туда все засасывает, то имеет ли белая дыра отрицательную массу?
Аноним 01/12/17 Птн 01:11:13  363319
>>363315
>имеет бесконечную массу
С хуя ли?
>то имеет ли белая дыра отрицательную массу
Нет.
Аноним 01/12/17 Птн 01:17:50  363320
>>363312
Полет союза раза в четыре дешевле (если еще не дешевле) шатола, плюс починка была за один полет. Дальше гиродины меняли и бачок омывателя подлили.
Когда речь идет о гигабаксовом телескопе то можно и не такие фортеля делать чтоб не топить попусту.
Аноним 01/12/17 Птн 01:35:54  363322
180px-Vonny.png (18Кб, 180x160)
>>363315
>черная дыра имеет бесконечную массу
птчк т упрлс?
Аноним 01/12/17 Птн 01:40:29  363324
>>363323
Самое дорогостоящее сооружение/устройство в мире построено роскосым и насой совместно.
Аноним 01/12/17 Птн 03:47:26  363329
medgallery69557[...].jpg (76Кб, 600x480)
>>363274
Черную дыру как такову увидеть невозможно, а вот преломление света звезд которые за ней - изи
Аноним 01/12/17 Птн 08:59:25  363341
>>363329
Я вижу черный круглешок.
Аноним 01/12/17 Птн 13:59:22  363370
>>363341
Ты видишь отсутствие фотонов потому что с обратной стороны они в неё воткнулись.
Аноним 01/12/17 Птн 14:16:25  363371
Что будет если попытаться выебать черную дыру? Допустим, я подлетаю в скафандре на все 12 сантиметров к горизонту событий, достаю и сую своего богатыря за горизонт. Что произойдет? Допустим, ЧД сверхмассивная и не вращается.
Аноним 01/12/17 Птн 14:31:09  363376
>>363371
Такой термоядерный минет даже пылесос не обеспечит.
Аноним 01/12/17 Птн 17:07:54  363382
>>363371
Останешься без хуя.
Аноним 01/12/17 Птн 17:32:13  363391
>>363382
Почему ведь основание останется по нашу сторону от горизонта?
Аноним 01/12/17 Птн 17:52:30  363397
>>363382
На самом деле нет.
Если он подлетит к ГС на 12 сантиметров, то за 0,4 наносекунды, он окажется полностью за ГС. Если же он будет лететь вдоль, то сила роршаха, распидорасит его тонким слоем по всему акреционному диску, и его хуй будет неотличим от любой другой его части. А т.к. эта масса анона неотличима по структуре, то возможно это интерпретировать как угодно. Но наиболее вероятная интерпретация, это то, что его хуй сплавился с его же телом. Т.е. попытавшись выебать ЧД ты выебешь сам себя.
Аноним 01/12/17 Птн 17:53:40  363399
>>363397
Он говорил про СМЧД, там градиент гравитации не достаточный, чтобы его размазать по аккреционному диску.
Аноним 01/12/17 Птн 18:52:01  363417
>>363391
Растянется.
Аноним 01/12/17 Птн 20:48:20  363441
>>363397
> выебешь сам себя.
Отлично, то что надо.
Аноним 02/12/17 Суб 01:59:43  363490
>>363441
Не проще ли тогда просто под ядераху подставиться?
Аноним 02/12/17 Суб 05:14:51  363501
Будет ли солнце приближатся к центру, как сильно, насколько вообще можно это предсказать?
Аноним 02/12/17 Суб 09:20:42  363519
>>363501
К какому центру?
Аноним 02/12/17 Суб 09:24:20  363520
15120267987320.jpg (44Кб, 676x774)
У меня вопрос по планетам. Какой высоты могут быть горы на планетах и чем они определяются? А ущелья насколько могут быть глубокими? А впадины (типа марианкой)?

И да, а что если Олимп на Марсе - не собственной марсианское "изделие", а упавший на поверхность предполагаемый третий спутник Марса?
Аноним 02/12/17 Суб 10:48:38  363525
>>363520
Высота гор растет до предела, обусловленного прочностью пород (R 200 МПА для одноосевого сжатия гранита и базальта без дефектов) и силой тяжести g на планете. При превышении этого предела начинает выдавливаться новый горный хребет, почти параллельный предыдущему - генерируется целая горная провинция. По этой причине высота гор на Земле (h) ни при каких условиях не может быть больше 14.8 км ~ 2 x 7.4 км (200 МПА > 0,5 x (g x d x h) = 0,5 x(9.8x2800x7400),где d - плотность мантии). Коэффициент 2 появляется из-за того, что горы не параллелепипеды, а, скорее, лежащие на боковой грани треугольные призмы с сечением S=0,5 xBxh. Поскольку реально высота гор от подножия до вершины (а не над уровнем моря) не превышают 5 км, мы должны сделать вывод, что эффективная прочность пород коры, по крайней мере, втрое меньше взятой из справочника (для бездефектного образца).
То есть, из-за различных дефектов в теле гор, а также из-за дополнительного сопротивления (сверх преодолеваемого литостатического давления выдавливаемых гор ) при их выдавливании из коры, горы на Земле никогда не достигают максимально возможной высоты (соответствующей прочности бездефектных пород). Даже под водой, где часть давления горы компенсируется давлением воды. Кстати, по этой причине подводные горы могут иметь несколько большую крутизну и высоту, чем горы на суше.
Аноним 02/12/17 Суб 12:54:17  363542
>>363519
Галактикусы.
Вопрос про орбиты.
Аноним 02/12/17 Суб 13:08:48  363545
>>363520
Про олимпус. Могу сказать один косвенный факт. Горная порода на олимпусе почти идентична породе на гаваях. Как образовались гаваи. Это зона вулканического плюма. Что есть структура начинающаяся от ядра, до поверхности. Подробнее можешь прочитать в интернете. Можно предположить что олимпус тоже плюм.
Аноним 02/12/17 Суб 15:40:35  363592
Вот я увидел, что размер магнитосферы земли почти совпадают с дистанцией до марса во время противостояний.
Дотягивается ли? Какова вероятность попадания этого хвостика в тушу планеты? Каковы были бы результаты? Те же вопрос с газовыми гигантами.

Другой вопрос, более ебанут. Кто нибудь запускал автомобиль в космос в качестве спутника?
Аноним 02/12/17 Суб 15:56:06  363596
>>363592
>Кто нибудь запускал автомобиль в космос в качестве спутника?
Нет. Потому, что нахуя? Самое ебанутое - запускали самолёт на орбиту, но у него маршевые водородные движки были и реактивная система управления.
Аноним 02/12/17 Суб 16:05:15  363602
>>363592
> Кто нибудь запускал автомобиль в космос в качестве спутника?
Маск в январе запустит
https://meduza.io/news/2017/12/02/ilon-mask-poobeschal-zapustit-falcon-heavy-v-yanvare-2018-go-raketa-otpravit-k-marsu-mashinu-tesla-c-pesney-space-oddity
Аноним 02/12/17 Суб 16:26:02  363605
>>363602
Да он ебанутый!
Аноним 02/12/17 Суб 16:29:50  363606
изображение.png (676Кб, 607x1024)
>>363605
Первый запуск очевидно рискованный, поэтому полезную нагрузку нет смысла запускать
Аноним 02/12/17 Суб 21:27:35  363665
>>363542
Орбита Солнца вокруг смчд Млечного Пути известна. Вроде почти круговая
Аноним 02/12/17 Суб 21:28:35  363666
>>363596
Можно подробнее про самолёт а космосе? Или ты про шатол?
Аноним 02/12/17 Суб 21:30:53  363668
>>363666
>шатол
this
Тащил с собой крылья на орбиту, которые там мертвый груз.
Из "крылатых" очень крут был бы венчурстар.
Аноним 02/12/17 Суб 21:33:22  363669
>>363665
Эта черная дырень довольно "маленькая". По крайней мере ее масса по сравнению со всей галактикой пренебрежимо мала (доли процента). А вот Солнце составляет более 99% массы своей системы, для сравнения.
Аноним 02/12/17 Суб 21:34:22  363670
>>363669
И что? Все равно всё в Млечном Пути вокруг нее крутится
Аноним 02/12/17 Суб 21:35:10  363671
space-artistic-[...].jpg (313Кб, 1920x1178)
>>363668
Эх, а я уже подумал про что-то уровня пикрила
Аноним 02/12/17 Суб 21:36:12  363673
image.png (137Кб, 300x225)
>>363671
Ладно хоть не это. Сцена из Лангольеров.
Аноним 02/12/17 Суб 21:38:34  363675
>>363670
Ну это спорный вопрос. Если ее убрать, будет крутиться ровно так же. В некотором смысле, это дырень образовалась в центре, а не центр там, потому что дырень. Ну или за миллионы лет центр и дырень постепенно совместились. А вот если убрать Солнце, вся система разлетится к хуям сражу зе.
Аноним 02/12/17 Суб 21:41:34  363677
>>363675
>Будет крутиться ровно также
И что же заставит миллиарды светил не двигаться по прямой?
Аноним 02/12/17 Суб 21:42:39  363678
>>363677
Притяжение друг к другу. Вероятно, еще и к черному веществу: в теории, его дохуя, а взаимодействует с обычным веществом только гравитационно.
Аноним 02/12/17 Суб 21:50:07  363683
>>363678
К темной материи, в смысле. Зобыл термин, лол.
Аноним 02/12/17 Суб 21:55:07  363685
Что современная наука говорит о теории что Меркурий это слетевший с орбиты спутник Венеры?
Аноним 02/12/17 Суб 22:31:27  363687
>>363685
Говорит, что это не так.
Аноним 03/12/17 Вск 11:25:13  363716
>>363673
Более трешаёбски наркоманскую концепцию времени и того, что с ним может происходить нигде не видел, до сих пор дёргает.
Аноним 03/12/17 Вск 11:56:47  363721
жi.jpg (10Кб, 320x240)
>>363716
А почему нет? Неужели ты боишься тех монстров?
Аноним 03/12/17 Вск 11:57:05  363722
>>363716
Степан Король же автор, чего ты хотел?
Аноним 03/12/17 Вск 13:05:24  363734
В треде роскосого в оппикчах есть схема 3 ракет.
Самая первая какая-то кривая.
Это ведь нихуя не стабильно, не так ли?
Аноним 03/12/17 Вск 14:30:10  363760
Какое масло способствует большему выделению канцерогенов подсолнечное, сливочное или оливковое?
Аноним 03/12/17 Вск 14:46:39  363762
>>363760
Тебе не кажется что ты di и spc перепутал?
Аноним 03/12/17 Вск 15:18:57  363763
>>363721
>>363722
Я могу на изи представить как в нашу реальность проникает подобная дичь и начинает жрать само пространство время оставляя на нём фрактальные узоры кристаллоидных полей, а через оставленные ими дырки видна БЕЗДНА ВЕЧНОСТИ откуда начинают лезть чёрные черви извергающие фонтаны экзочастиц похожие на метафизическую кровищу с говнищем, а по всей вселенной начинает раздаваться наипротивнейший телепатический зов в стиле НИЧТО ЭТО СМЫСЛ. ПУСТОТА ЭТО ВЕЧНОСТЬ. ПРИМИ НАС, НЕБЫТИЕ ЛЮБИТ ТЕБЯ, ПУСТОТА ЭТО ИСТИНА, БОГ ЭТО МЫ. Потом дырки начинают цвести фазовыми распадами вакуума из за ебанистической иной физики, один из таких ВНЕЗАПНО накрывает земляшку, но никто не умир, солнце стало странной черной дырой жуткого вида как будто её прогнули через несколько измерений и завязали узлом короче типа черная дыра с "волосами" или как то так всё остальное пространство покрылось нитями с которых выделяется тёмно красно коричневая экзотина, по всей земле ползают червяки, которых пытаются сожрать те йоба монстры, а те их пытаются сожрать в ответ. Но если от кого чё откусывают то тут же новое отрастает, если сожрут целиком то высрут в целом же виде ну или не совсем, не целое отрастёт по быстрому. Сожрут если человеков\земляшко животинку тоже отрастёт но больно пиздец, и такая хуйня продолжает миллиарды триллионов лет, все пытаются друг друга сожрать, но не хрена не получается и всё это под постоянную телепатическую музыку: НИЧТО ЭТО СМЫСЛ. ПУСТОТА ЭТО ВЕЧНОСТЬ. ПРИМИ НАС, НЕБЫТИЕ ЛЮБИТ ТЕБЯ, ПУСТОТА ЭТО ИСТИНА, БОГ ЭТО МЫ.

И даже при большем желании написать графоманию на эту тему, так что не думаю что прямо боюсь, но если бы рили сдох от такого или хотя бы увидел словил бы тот ещё бсод. Впрочем наверное пора уже привыкнуть ожидать от реальности всё что я и вообразить то при всём желании не смогу, нет же абсолютных гарантий что она останется такой какой мы её знаем на вечность вечную...
Аноним 03/12/17 Вск 15:45:19  363765
>>363763
14000000000 лет оставалась, а тут изменится? Шанс есть, но на твоём веку он равен в лучшем случае 1/140000000. Ну это если такая херонина не происходила больше нигде во вселенной, а если предположить, что происходила, но мы этого в телескопы разглядеть не можем, то шанс уже повыше, но всё равно крайне мал ибо в нашем например сверхскоплении за 14млрд лет ни разу не произошло.
Аноним 03/12/17 Вск 16:04:32  363773
>>363763
Нас Тебя скорее гамма всплеском выжарит или случайная обожэмой частица прицельно по мозгам долбанёт или же Солнце выбросит столько плазмы, что хватит тебя по быстрому расплавить, чем какая нибудь параша с физикой случится особенно прямо такая дичь уровня \b\ вроде той что ты описал но да абсолютной 146% гарантии что все будет так как было прямо аш всегда никто не давал. Учитывая что например Большой бабах всё таки был 14ккк лет назад, а не вселенная существует уже хрен знает сколько лет.
Аноним 04/12/17 Пнд 00:36:32  363832
Господа, вопросотред уже почти ушёл с нулевой, в связи с чем вопрос: вы охуели?
Аноним 04/12/17 Пнд 00:46:46  363833
>>363832
Возьми и перекати, нытик.
Аноним 04/12/17 Пнд 10:32:45  363845
>>363832
Дрочую. И ссыль на новый тред мне запилите блять! А то как обычно буду тут сидеть один аутировать, в мне развивается надо. Делайте бляди быстрее уже.
перекатить не могу, сижу с тапка
Аноним 04/12/17 Пнд 10:51:00  363846
>>363734
Вот ПРОКЛЯТЬЕ. Гуглил-гуглил и не могу найти пикчу что раньше видел.
Там был Атлас 5, японская Хэ-2, Шаттл и чето еще.
Несимметричные ракеты бывают.
ПЕРЕКАТ Аноним # OP  04/12/17 Пнд 10:56:48  363849
Перекат:
>>363848 (OP)
>>363848 (OP)
>>363848 (OP)
Аноним 18/12/17 Пнд 18:20:13  365722
Анончик, расскажи плиз о квантовых эффектах - какие бывают?

Что такое квантовая запутанность как вот прямо берут пинцетиком два фотона и путают?

Как осуществляется квантовая связь? - по идее один из фотончиков должны отнести пинцетиком на другой конец планеты - на практике обычное оптоволокно

все статьи в интернете - куча рекламных общих слов нихуя конкретики и пахнет все пиздежом(((


Топ тредов
Избранное